Формула тормозной путь автомобиля: Тормозной путь автомобиля. Формула расчёта тормозного пути

Содержание

Как правильно вычислить дистанцию, тормозной и остановочный путь автомобиля: формулы расчета

Формулы расчета остановочного и тормозного пути, а также безопасной дистанции.

В теоретическом экзамене есть вопрос о среднем времени реакции водителя, правильным ответом на который является 1 секунда. Также в билетах ГИБДД имеется вопрос, связанный с безопасной дистанцией. Есть вопросы, касаемые торможения. Но, как говорится, теория – это теория, которая, увы, с практикой, как правило, не имеет ничего общего.

Во-первых, то, что вы учили в билетах, является теорией, основанной на усредненных значениях и различных исследованиях. Фактически же время реакции водителя, остановочный и тормозной путь зависят от многих факторов и не могут быть точно рассчитаны для всех случаев. Тем не менее каждый водитель должен уметь рассчитывать эти параметры хотя бы приблизительно. 

 

Тормозной путь автомобиля

Тормозной путь – это расстояние, которое будет пройдено автомобилем между контактом водителя с педалью тормоза и полной остановкой транспортного средства. Также стоит понимать различия между «нормальным торможением» и «экстренным торможением». В том числе не нужно забывать, что погодные условия влияют на тормозной путь. Если на дороге есть снег, тормозной путь, естественно, увеличивается. 

 

 

Вот формула расчета тормозного пути:

 

(Скорость в км / ч: 10) x (скорость в км / ч: 10) = тормозной путь в метрах

 

Пример расчета: представим, что вы едете со скоростью 50 км/ч по городу и подъезжаете к пешеходному переходу, по которому идут дети. Расчет: (50 км/ч : 10) х (50 км/ч : 10) = 25 (метров). Таким образом, тормозной путь вашей машины составляет 25 метров. Поэтому вы должны учитывать длину тормозного пути, чтобы спокойно своевременно начать тормозить и остановиться перед пешеходным переходом. 

 

Имейте в виду, что при экстренном торможении вы обычно нажимаете педаль тормоза полностью. В этом случае, как правило, тормозной путь сокращается вдвое. Вот формула тормозного пути при экстренном торможении:

 

 

(Скорость в км / ч: 10) x (скорость в км / ч: 10) / 2 = тормозной путь в метрах 

 

Пример расчета: вы едете по городу со скоростью 50 км/ч, и вдруг на дорогу выкатывается мяч, за ним бежит ребенок. Вам нужна экстренная остановка автомобиля. Расчет: (50 км/ч : 10) х (50 км/ч : 10)/2 = 12,5 (метров). Тормозной путь вашей машины при экстренном торможении составит 12,5 метра. 

 

 

Время и путь реакции водителя

Время реакции водителя – это время, которое пройдет с момента обнаружения водителем опасности на дороге до начала принятия мер по ее предотвращению.

 

Путь реакции водителя – это путь, который пройдет автомобиль с момента обнаружения водителем опасности на дороге до нажатия педали тормоза. 

Вот формула расчета пути, который пройдет автомобиль в момент реакции водителя на опасность: 

 

 

(Скорость в км / ч: 10) x 3 = путь реакции в метрах

 

Пример расчета: представим, что вы едете со скоростью 100 км/ч по проселочной дороге и внезапно на дорогу выбегает лось. Расчет: (100 км/ч : 10) х 3 = 30 (метров). То есть, после того как вы среагируете на опасность на дороге, ваша машина проедет примерно 30 метров. Добавьте к этому тормозной путь автомобиля.

 

Внимание: эти правила не являются научно правильными формулами и дают только приблизительное значение!

 

Остановочный путь автомобиля

Остановочный путь – это расстояние, пройденное транспортным средством с момента обнаружения водителем опасности на дороге до полной остановки машины. 

 

 

Если вы хотите рассчитать остановочный путь автомобиля, вы должны добавить к тормозному пути автомобиля путь, пройденный за время реакции водителя. Вот как это можно сделать:

 

(Скорость в км / ч: 10) х 3 + (скорость в км / ч: 10) х (скорость в км / ч: 10)

Первое значение в выражении – это путь реакции водителя, пройденный автомобилем, пока водитель реагирует на опасность на дороге. Второе выражение – это формула расчета тормозного пути. Для того чтобы вычислить остановочный путь транспортного средства, необходимо оба результата сложить вместе. 

 

Пример расчета:  вы едете на своей машине со скоростью  50 км/ч.

Расчет: (50 км/ч : 10) х 3 = 15 метров пути проедет машина при реагировании на опасность на дороге (50 км/ч : 10) x (50 км/ч : 10) = 25 метров составит тормозной путь автомобиля. В итоге, сложив оба значения, получаем, что остановочный путь транспортного средства составит 40 метров. 

 

Внимание: эти правила не являются научно правильными формулами и дают только приблизительное значение!

 

Дистанция

  • Три длины автомобиля. Любой, кто путешествует в городских условиях, должен соблюдать дистанцию ​​не менее 15 метров, или три длины автомобиля.
  • Половина спидометра: для безопасной дистанции за пределами населенных пунктов обращайте внимание на скорость автомобиля. Для того чтобы вычислить безопасную дистанцию, разделите на 2 текущую скорость, которую показывает спидометр. В итоге вы получите дистанцию до других автомобилей в метрах. Пример: на скорости 70 км/ч вы должны держаться до впереди идущего автомобиля на расстоянии не менее 35 метров. Причем это касается сухого асфальта в летнее время. 
  • Двойное расстояние: в случае плохой видимости или плохих дорожных условий вы должны удвоить безопасную дистанцию.

Вот как можно рассчитать тормозной путь: Формула

Как рассчитать расстояние тормозного пути автомобиля.

 

Как быстро автомобиль ускоряется, наверное, знает большинство автовладельцев. Даже если вы не замеряли динамику разгона своей машины, вы наверняка смотрели заводские технические характеристики вашего авто, где обычно автопроизводитель указывает минимально возможное время разгона с 0-100 км/час. Но теперь вопрос: сколько времени нужно, чтобы остановить вашу машину? Вы знаете это? Уверены, что нет. Но, оказывается, рассчитать расстояние тормозного пути можно достаточно легко с помощью простой формулы. Мы расскажем вам, как это делается. 

 

Нет такой вещи во Вселенной или материи, которая может мгновенно остановиться. Также и любой автомобиль, когда вы нажимаете педаль тормоза, не сразу может остановиться. Дело в том, что для того чтобы автомобиль или любой объект в нашем мире остановился, необходимо, чтобы он потерял энергию, которая его движет. В результате у любого автомобиля есть тормозной путь, который он проезжает с момента нажатия педали тормоза до момента полной остановки. Это и есть тормозное расстояние машины.

 

Но на самом деле тормозной путь любого авто зависит не только от его характеристик и тормозной системы, но и от реакции водителя при нажатии педали тормоза. Ведь для того чтобы принять решение о необходимости торможения и нажать педаль тормоза, требуется время, которое хоть и минимально, но достаточно, чтобы машина успела проехать немаленький путь. Особенно это важно при большой скорости движения, где за какие-то доли секунды автомобиль проезжает приличное расстояние. Итак, в итоге, чтобы рассчитать реальную длину тормозного пути, нужно учитывать не только время и расстояние, пройденное автомобилем с момента нажатия водителем педали тормоза до момента остановки машины, но и время, необходимое для принятия решения о торможении. Дело в том, что при принятии решения о торможении мы тратим драгоценные секунды. Вот пример:

 

  • Время отклика: Прежде чем водитель нажмет педаль тормоза, он должен оценить дорожную ситуацию и определить, необходимо ли торможение. Также нужно понять, какое необходимо торможение – полная остановка автомобиля или простое снижение скорости. Обычно, согласно многочисленным исследованиям, большинству водителей для этого требуется около 0,1 секунды. 
  • Время, необходимое для нажатия педали тормоза: После того, как водитель понял, что должен тормозить, необходимо еще примерно 0,8 секунды, для того чтобы переместить ногу с педали газа на педаль тормоза и нажать ее. 

 

Кроме того, даже при нажатии педали тормоза есть еще небольшая потеря времени, связанная с тем, что при нажатии педали тормоза автомобиль, как правило, не начинает резко тормозить. А для того чтобы машина реально начала резко снижать скорость, надо усилить давление на педаль тормоза (пороговое время, необходимое для требуемого тормозного давления в тормозной системе). Также у всех автомобилей разное время отклика на нажатую педаль тормоза. Здесь все, конечно, зависит от конструкции тормозной системы и наличия различной электроники, контролирующей тормоза автомобиля.

 

Смотрите также: Полный привод оказался лучше при торможении, чем привод на два колеса: Видео

 

Вы не поверите, но для того чтобы машина реально начала тормозить после нажатия педали тормоза, необходима еще почти 1 секунда времени. Вы представляете, как это много при движении на большой скорости? За эту лишнюю секунду вы можете проехать очень большой путь. 

 

Что такое формула тормозного пути?

В общем, торможение автомобиля делится на два вида. Например, есть нормальное торможение, а есть экстренное, когда вам нужно резко остановить машину, чтобы избежать аварии.

 

При торможении в повседневной жизни, допустим, если вы хотите остановить автомобиль на светофоре, вы обычно нажимаете педаль тормоза намного плавнее и мягче, чем при необходимости полностью остановить автомобиль на парковке во дворе. В этом случае вы не применяете в машине максимальное тормозное усилие. При таком плавном и мягком торможении, как правило, тормозной путь (тормозное расстояние) увеличивается. Примерное расстояние тормозного пути при нормальном торможении можно рассчитать по следующей простой формуле:

 

(Скорость в км/ч : 10) x (скорость в км/ч : 10) = тормозной путь в метрах

 

При экстренном торможении педаль тормоза, как правило, нажата целиком и с полной силой. Из-за более высокой силы торможения обычно тормозной путь машины сокращается примерно в 2 раза. Поэтому длину тормозного пути можно также вычислить по следующей формуле:

 

(Скорость в км/ч : 10) x (скорость в км/ч : 10) / 2 = тормозной путь в метрах

 

Внимание: Вычисляемый по этим формулам тормозной путь является лишь приблизительным значением и подсказкой для водителей. На самом деле в реальности тормозной путь может быть как меньше, так и больше. Ведь расстояние тормозного пути зависит от навыков и опыта вождения водителя, от технической исправности автомобиля, его конструкции, марки, модели, состояния дорог, состояния протектора резины и многих других факторов, которые напрямую влияют на длину тормозного пути. Но благодаря этим формулам вы примерно сможете высчитать среднюю длину тормозного пути машины при определенной скорости движения. Это позволит вам скорректировать ваш стиль управления автомобилем, а также станет хорошим пособием для водителей-новичков. 

 

Как рассчитать полное время остановки и итоговый тормозной путь?

 

Как мы уже сказали, чтобы рассчитать весь тормозной путь, нужно учитывать потерю времени при принятии водителем решения о торможении (то есть время реакции водителя). Для этого нужно использовать другую формулу, которая обеспечивает более точный приблизительный расчет тормозного расстояния, которое проедет автомобиль в момент принятия решения о необходимости остановки. Вот эта формула:

 

(Скорость в км/ч : 10) x 3 = путь реакции в метрах

 

В итоге, сделав вычисление по вышеуказанным формулам, вы можете вычислить приблизительный итоговый тормозной путь вашего автомобиля при любой скорости движения. Вот пример. Если вы управляете своим автомобилем со скоростью 50 км/ч, то с помощью приведенных формул вычислите следующие значения:

 

  • Тормозной путь при принятии решения о торможении на этой скорости (реакция на дорожную ситуацию + принятие решения о торможении + время, необходимое для перемещения ноги с педали газа на педаль тормоза, а также время отклика тормозной системы на нажатую педаль тормоза) составит где-то (50/10) х 3 = 15 метров. То есть пока вы будете принимать решение о торможении при скорости в 50 км/ч, ваша машина проедет 15 метров. 
  • Тормозной путь при нормальном торможении (с момента нажатия педали тормоза до момента остановки машины) составит около (50/10) х (50/10) = 25 метров. 
  • При экстренном торможении тормозной путь, как мы уже отметили, сокращается примерно в два раза. Соответственно, расчет тормозного расстояния автомобиля, который движется со скоростью 50 км/ч, будет выглядеть следующим образом: (50/10) x (50/10) / 2 = 12,5 метров.
  • В результате теперь мы можем вычислить реальный итоговый тормозной путь автомобиля. Так, при нормальном (не резком, а обычном) торможении итоговый тормозной путь составит около 40 метров. При экстренном торможении – не менее 28 метров. 

 

Примечание: Обратите внимание, что если скорость автомобиля будет выше всего в два раза, его итоговый тормозной путь увеличится в четыре раза!!!

 

Смотрите также: Основные принципы работы тормозного механизма автомобиля [Принцип работы и элементы тормозной системы]

 

То есть мнение о том, что при увеличении скорости автомобиля в два раза тормозной путь увеличивается только в два раза, – это чистый воды миф среди многих автолюбителей. Так что имейте это в виду, когда садитесь за руль. Самое удивительное, что об этом не знают даже многие опытные водители. 

 

Пример расчета тормозных и остановочных расстояний

Скорость, в км / ч

Путь, пройденный автомобилем

во время реакции водителя, в метрах

Тормозное расстояние, в метрах

(с момента нажатия педали тормоза

до полной остановки машины)

Итоговый тормозной путь, в метрах

25

7,5

6,25

13,75

50

15

25

40

100

30

100

130

150

45

225

265

200

60

400

460

 

Какие факторы влияют на торможение и тормозной путь?

 

Решающим значением для длины тормозного пути, конечно же, является скорость автомобиля, с которой он движется по дороге. Также на тормозной путь влияет качество установленной на машину тормозной системы. В том числе важную роль, несомненно, играет и состояние дороги (снег, лед, качество асфальта/бетона, трещины в дорожном покрытии, листья, лужи и т. п.). И само собой, не стоит забывать о состоянии шин автомобиля. Ведь в определенных случаях изношенная резина сильно увеличит тормозной путь автомобиля, так как не сможет передавать нормальную тормозную способность дорожному покрытию в отличие от новых шин, имеющих нормальное сцепление с дорогой. 

Также ясно, что на мокрой поверхности тормозное расстояние машины больше, чем на сухом асфальте. 

 

Не стоит забывать и об уровне подготовки водителя. Особенно важна, как мы узнали, для итогового тормозного пути скорость реакции водителя на дорожную ситуацию, требующую остановки автомобиля. Но скорость реакции за рулем зависит не только от опыта вождения. Например, знаете ли вы, что когда вы садитесь за руль в сонном состоянии (не выспались, устали или долго находились за рулем), то скорость реакции может замедлиться почти в два раза по сравнению со скоростью реакции хорошо отдохнувшего водителя. 

В целом же на скорость принятия решения за рулем (скорость реакции) влияет много факторов: возраст водителя, алкогольное или похмельное состояние, употребление определенных медикаментов и в целом состояние здоровья. Так, при многих хронических заболеваниях скорость реакции многих водителей существенно снижается. Следовательно, все эти факторы серьезно влияют на тормозной путь автомобиля. 

 

Смотрите также: Тормозной путь автомобиля: Все что нужно знать

 

То же самое касается и отвлечения внимания из-за смартфонов, которыми так любят пользоваться за рулем многие водители, несмотря на строгий запрет согласно нашему действующему законодательству.

 

Как мы уже сказали, на тормозной путь также влияет время отклика тормозной системы автомобиля на нажатую педаль тормоза. Особенно это касается старых автомобилей. Современные же, как правило, оснащены уже новым поколением тормозов, которые мгновенно активируются за счет максимального тормозного давления, как только вы резко ударите ногой по педали тормоза (например, при экстренном торможении). Эта технология позволила существенно сократить итоговый тормозной путь современных машин. 

 

Как повысить безопасность при управлении автомобилем?

 

Не зря основное правило вождения гласит о том, что водитель должен держать на дороге достаточную дистанцию до других автомобилей, чтобы оставалось пространство для экстренного торможения и для того, чтобы не спровоцировать ДТП. Но, с другой стороны, вы не должны держать дистанцию между автомобилями слишком большой. Помните, что все должно быть в меру. Вот некоторые правила вождения от экспертов:

 

  • В городском движении: Держите расстояние до других автомобилей около 15 метров. 
  • На автомагистралях, шоссе и проселочных дорогах: При скорости движения около 100 км/ч держите дистанцию примерно 50 метров. При плохой видимости или на скользкой дороге дистанция до других машин должна быть увеличена в два раза. Например, при скорости в 100 км/ч на скользкой дороге держите расстояние до впереди идущей машины минимум в 100 метров. 

Тормозной путь автомобиля

Длину тормозного пути своего автомобиля должен знать каждый водитель. Особенно важно понимать и осознавать, что длина торможения разнится не только от статуса авто, но и от вида дорожного покрытия и времени года.

Длина тормозного пути автомобиля

Длина тормозного пути автомобиля в зависимости от нескольких факторов может составить от 25 до 150 метров. Длина зависит от способности конкретной модели авто сбрасывать скорость до нужного показателя, включая остановку, и при этом оставаться устойчивым и управляемым. Теоретически для оценки тормозных характеристик авто применяют несколько показателей: тормозной путь, максимальное значение замедления, время срабатывания механизмов торможения, диапазон изменения усилий торможения, уменьшение эффективности торможения из-за сильного нагрева.

Расчёт тормозного пути автомобиля

Расчет тормозного пути автомобиля возможен по нескольким формулам. Теоретически длина автомобиля – это не что иное, как зависимость следующих величин: Sт = Vн х tср + Vн2 / 2aт. Умножение начальной скорости движения на время срабатывания тормозной системы, где aт – это замедление хода автомобиля.

Чтобы определить максимальное замедление ТС, нужно воспользоваться следующей формулой: amax=g*µhf, в которой g – это ускорение свободного падения, а под µhf подразумевается коэффициент сцепления шин с дорогой.

Тормозной путь легкового автомобиля

Тормозной путь легкового автомобиля – это расстояние, пройденное им с того момента, как произошло нажатие на педаль тормоза до полного прекращения вращения колес. От чего зависит метраж тормозного пути? Немаловажный фактор – это время года: зимой, когда на дорогах гололед, тормозной путь куда длиннее, и может составить около 100-150 метров. А вот летом, в жару, наоборот, не более 25-30 метров. Но все зависит и от конкретной марки авто и ее личных характеристик. Также коэффициент сцепления шин с дорогой зависит, собственно говоря, от самого дорожного полотна и качества шин.

Совет от Сравни.ру: как говорится, тише едешь – дальше будешь. Если вы планируете поездку в сильный снегопад или гололед, помните, в случае резкого торможения ваше авто не станет, как вкопанное моментально. Будьте внимательны за рулём.

Остановочный путь

Остановочный путь

Путь реакции

Путь реакции это расстояние, которое автомобиль успел проехать с того момента, как вы заметили опасность, до того момента, как вы начали тормозить или поворачивать.

Путь реакции зависит от:
  • Скорости автомобиля. Путь реакции прямо пропорционален скорости: в 2 раза больше скорость = в 2 раза больше путь реакции.
  • Времени реакции. Нормальное время реакции человека составляет 0.5 — 2 сек. На время реакции оказывают влияние опыт, возраст, состояние водителя и многие внешние факторы. Обычно лучшее время реакции у опытных водителей, в возрасте 45 — 54 года.
Путь реакции можно существенно сократить, если вы:
  • Предвидите опасную ситуацию.
  • Внимательно следите за дорогой и готовы к действиям.
Путь реакции может существенно увеличиться, если
Расчет пути реакции

Допустим, автомобиль движется со скоростью 50 км/час и время реакции составляет 1.5 секунды.

  • Переводим км/час в м/с. 50 + 10 % = 55
    55 / 4 = 13.75 м/с
  • Умножаем скорость (в м/с) на время (в сек.) получаем пройденный путь. 13.75 * 1.5 путь реакции = 20.625 метра.

Тормозной путь

Тормозным путем называется расстояние, которое проезжает автомобиль с момента начала торможения и до полной остановки.

Тормозной путь зависит от:
  • скорости автомобиля, рост квадратичный, в 2 раза больше скорость => в 4 раза больше тормозной путь. в 3 раза увеличивается скорость => в 9 раз возрастает тормозной путь.
  • состояния дороги, играют роль уклон, состояние дорожного покрытия, сухая дорога или мокрая и пр.
  • массы автомобиля, у груженого автомобиля тормозной путь больше.
  • колес и тормозов, состояние тормозной системы, количество колес, качество протектора, наличие дополнительных систем торможения и пр.
Расчет тормозного пути

Очень трудно расчитать тормозной путь для абстрактного автомобиля. Обычно большинство задач сводится к тому, что зная тормозной путь на одной скорости, необходимо вычислить его для другой скорости. Зная, что зависимость квардратичная, это достаточно просто. Тем не менее есть некоторые цифры, которые можно брать за основу.

Считается, что средний автомобиль на хорошей сухой дороге, двигаясь со скоростью 10 км/час, имеет тормозной путь 0.4 метра. Соответственно, для скорости 20 км/час он составит 1.6 метра, 30 км/час — 3.6 метра, 50 км/час — 10 метров.

Более точные цифры можно получить, воспользовавшись формулой S = V² / (250 * k), в которой S это тормозной путь, V — скорость автомобиля в км/час, k — коэффициент трения колес по асфальту (0.8 для сухой дороги — 0.1 для льда). Формула дает результат для скорости 50 км/час — 12.5 метров.

Остановочный путь

Остановочный путь есть сумма пути реакции и тормозного пути. Задачи по вычислению остановочного пути сводятся к вычислениям пути реакции и тормозного пути.

Обычно в экзаменационных вопросах разница между вариантами ответов достаточно существенна. Вам не нужно вычислять подобные цифры с точностью до знака после запятой. Если приближенное вычисление показывает ответ «12», то, как правило, этого достаточно, если вам необходимо выбрать между вариантами ответов «5», «10» и «20».

Учебник ПДД | Содержание

Как рассчитать тормозной путь автомобиля

Каждый водитель хоть раз да оказывался буквально в паре секунды от аварии, когда жизненно необходимо успеть затормозить. Однако встать, как вкопанный по команде автомобиль не может. Расстояние, которое он проедет с момента начала торможения до полной остановки и называют тормозным путём. Уметь прикинуть тормозной путь нужно, чтобы он всегда был меньше, чем расстояние до оказавшейся на пути помехи.

Длина пути торможения зависит от множества разных факторов. Тут и реакция водителя, и уровень работы тормозной системы автомобиля, и внешние факторы, вроде материала трассы и погодных условий. Ну и конечно, решающую роль играет скорость машины на момент торможения. Появляется вопрос — как рассчитать тормозной путь автомобиля при всех этих условиях? Для общих расчётов достаточно трёх главных факторов — тормозного коэффициента (Кэ), скорости движения (V) и коэффициента сцепления (Фс) с трассой.

Формула для расчёта тормозного пути автомобиля

Формула из таблицы, вычисляющая длину тормозного пути, выглядит так: S=Кэ*V*V/(254*Фс). Тормозной коэффициент у обычного легкого автомобиля равняется единице. Коэффициент сцепления на сухой поверхности будет равен 0,7. Для примера, возьмём случай, когда машина движется по сухой трассе со скоростью в 60 км/ч. Тогда длина тормозного пути будет равна 1*60*60/(254*0,7)=20,25 метра. На льду же (Фс=0,1) торможение продлится в семь раз дольше — 141,7 метра!

По результату видим, как сильно длина тормозного пути автомобиля из таблицы зависит от состояния трассы и погодных условий.

Торможение в разных условиях

Длина тормозного пути обратно пропорциональна коэффициенту сцепления с трассой. Проще говоря — чем хуже “держит” дорога, тем дольше машина тормозит. Посмотрим на изменения коэффициента (Фс) подробнее:

  • при сухом асфальте — 0,7;
  • на мокром асфальте — 0,4;
  • если укатан снег — 0,2;
  • обледеневшая дорога — 0,1.

Эти цифры позволяют нам увидеть, как изменится тормозной путь в зависимости от условий. Как уже говорилось, при скорости 60 км/ч на сухой дороге автомобиль будет тормозить 20,25 метра, а на льду — 141,7. На мокрой трассе дистанция торможения составит 35,4 метра, а на заснеженной — 70,8.

Типы торможений

Типы торможения

Стоит также учитывать, что большую роль играет способ торможения:

  1. Резкое нажатие может отправить автомобиль в неконтролируемый занос.
  2. Постепенное нажатие на педаль сработает при хорошей видимости и запасе времени, но его не применить в экстренной ситуации.
  3. Прерывистое торможение с несколькими нажатием на педаль до упора позволит быстро остановить машину, но также чревато потерей контроля.
  4. Ступенчатое нажатие позволит блокировать колёса, не потеряв контакт с педалью.

Торможение с ABS

Система ABS работает как раз по принципу ступенчатого торможения, а её основная задача — не отпустить машину в неконтролируемый занос. ABS не блокирует колёса полностью, тем самым оставляя водителю контроль над движением автомобиля. Обильные проверки показали, что ABS сократит тормозной путь на сухом или мокром асфальте, а также отлично работает на гравии. А вот в других условиях система частично теряет свою ценность.

В зимних условиях ABS увеличит тормозной путь на 15-30 метров при движении по снегу или льду. При этом система оставит водителю контроль над машиной, что может быть критически важно при движении по гололёду.

Таблица трения при разных скоростях

Помните, слабых места ABS — раскисшая земля и глина. На них тормозной путь также может стать дольше, чем при полностью “ручном” торможении. Но и контроль над машиной также останется.

Как определить скорость автомобиля по тормозному пути?

В тех случаях, когда затормозить вовремя всё же не удалось, необходимо определить, с какой скоростью двигался транспорт на момент начала торможения. Общая формула вычисления “стартовой” скорости торможения выглядит так — V = 0,5*t3*j + √2*S*j. В данном случае, роль играют следующие факторы:

  • — время нарастания замедления машины. Измеряется в секундах;
  • j — замедление автомобиля при торможении. Измеряется в м/с2. По ГОСТу на сухой трассе j=6,8 м;
  • с2, а на мокрой — 5 м/с2;
  • S — длина тормозного следа.

Возьмём условия, в которых tЗ=0,3 секунды, тормозной след 20 метров, а трасса сухая. Тогда скорость равна 0,5*0,3*6,8 + √2*20*6,8 = 1,02 + 19,22 = 20,24 м/с = 72,86 км/ч.

В основном для определения скорости в начале торможения используются три способа:

  1. Определение по тормозному пути.
  2. Определение по закону сохранения количества движения.
  3. Определение по деформации автомобиля.

ABS, EBD и BAS

Преимущества первого метода — простота и скорость, большое количество исследований, точный результат. Второй метод хорош тем, что его можно использовать при отсутствии следов торможения, он даёт точный результат и полезен при столкновении с неподвижными машинами. Третий отличается тем, что учитывает энергозатраты на деформацию машины.

Минусы у каждого метода также свои. В первом случае это невозможность использования при отсутствии следов шин. Во втором — громоздкие вычисления, а в третьем — большие объёмы того, что нужно учесть, и невысокая точность вычислений.

Интересное по теме:

загрузка…

Facebook

Twitter

Вконтакте

Одноклассники

Google+

водителям пояснили / Авто / Судебно-юридическая газета

Для остановки автомобиля важны не только шины, но и время реакции водителя.

В автошколах будущих водителей учат, что общий тормозной путь рассчитывается по двум факторам — тормозной путь и время реакции.

Время реакции — это время, которое требуется водителю, чтобы распознать препятствие на дороге и среагировать на него — задействовать тормоза. Тормозной путь — это отрезок между реакцией водителя (торможением) и полной остановкой автомобиля.  

Какова формула для расчета пройденного расстояния во время реакции?

Время отклика, расстояние в метрах = (скорость/10) х3

Для среднестатистического водителя время отклика составляет около одной секунды. Если мы воспользуемся формулой, то узнаем, какое расстояние преодолеет автомобиль с одинаковой скоростью, прежде чем он начнет останавливаться.

Время отклика (расстояние) на скорости 50 км/ч = (50/10) x3 = 15 метров

Формула для расчета тормозного пути

Одна из городских легенд гласит, что для расчета тормозного пути достаточно разделить скорость на две части. Это неправильно по простой причине. Тормозной путь рассчитывается по приблизительной формуле, которая гласит:

Тормозной путь в метрах = (скорость/10) x (скорость/10)

Отсюда следует, что при удвоении скорости тормозной путь не удваивается, а умножается в четыре раза. Вот несколько примеров расчета тормозного пути на скоростях от 30 до 60 км/ч.

Тормозной путь при 30 км/ч = (30/10) x (30/10) = 9 метров

Тормозной путь при 50 км/ч = (50/10) x (50/10) = 25 метров

Тормозной путь при 60 км/ч = (60/10) x (60/10) = 36 метров

Формула для расчета общего тормозного пути

Для расчета общего времени торможения, включая расстояние, пройденное за время реакции, и расстояние остановки, эти два значения просто необходимо «собрать».

Общий тормозной путь при 30 км/ч = 9 + 9 = 18 метров

Общий тормозной путь при 50 км/ч = 15 + 25 = 40 метров

Общий тормозной путь при 60 км/ч = 18 + 36 = 54 метра

Напомним, ранее «Судебно-юридическая газета» сообщала, что составлен список самых красивых ретро-авто. 

Еще автоэксперты составили список самых экономически выгодных и доступных автомобилей на бензиновом топливе.

Также в столице увидели автомобиль с забавным номером.

Помимо этого, киевляне заметили элитную «евробляху».

Подписывайтесь на наш Telegram-канал, чтобы быть в курсе самых важных событий.

Как рассчитать остановочный путь автомобиля. Тормозной путь автомобиля

Беспечность выглядит эффектно только в хорошо продуманных сценах из боевиков и детективов. На самом же деле, большинство водителей даже не представляют, о какой опасности идёт речь, когда говорят о соблюдении дистанции и о превышении скорости. Многие ли падали с трехметровой высоты плашмя на бетонный пол? Едва ли. А на самом деле, точно такую же нагрузку будет испытывать человек в автомобиле при наезде на неподвижное препятствие на скорости… всего 28 км/ч.

Зачем знать длину тормозного пути

Раз уж мы начали с расчётов, говоря о длине тормозного пути движущегося автомобиля, используем простую физическую формулу, известную каждому школьнику. Её используют для вычисления перехода энергии падения в кинетическую энергию конце пути (mgh=mVx2/2). Отсюда получаем, что при скорости около 30 км/ч тело получает удар, равный падению с высоты три метра. Соответственно, при движении на скорости 60 км/ч сила удара будет равна падению с высоты 15м, а уже на скорости 90 км/ч — с высоты около 32 м, 120 км/ч — это уже высота 55 метров.

Даже учитывая, что в автомобиле срабатывает подушка безопасности, выжить при лобовом ударе на скорости 60 км/ч шансов очень мало. Это примерная высота хрущевки. Отважится ли кто-то прыгнуть с крыши пятиэтажки, обвязавшись надувными подушками? Едва ли. А что говорить о скорости в 90 км/ч, удар при которой равносилен падению с высоты десятиэтажного дома? А с высоты 55 метров? Шансов выжить никаких, и это даже при условии, что подушка безопасности сработает безукоризненно.

Эмпирическая формула расчёта тормозного пути

Имея отличный водительский глазомер и достаточный опыт, каждый сможет определить расстояние до объекта на глаз, хотя бы примерно. Водительский опыт показывает, что для мгновенного вычисления длины тормозного пути по скорости, необходимо просто бросить взгляд на спидометр, оценить расстояние до препятствия, тогда тормозной путь будет равен половине числа, которое показывает спидометр. То есть, исходя из эмпирической формулы расчёта длины тормозного пути, безопасная дистанция до любого объекта будет равна мгновенной скорости, разделённой пополам. Практически так же производят расчёт скорости автомобиля по тормозному пути.

При этом нужно учитывать такое понятие, как остановочный путь, это термин экспертов дорожной полиции и он учитывает не только сам по себе тормозной путь, но и скорость реакции, а также время реагирования системы тормозов. В принципе — это расстояние до абсолютной остановки машины от того момента, когда водитель зафиксировал препятствие. Естественно, остановочный путь всегда больше тормозного, поскольку средняя скорость реакции здорового и трезвого водителя около 0,8 с, а тормозная система срабатывает ещё за 0,2-0,3 с. Следовательно, до полной остановки машины пройдёт ещё 1,1 с, а на скорости 60 км/ч автомобиль проходит 16,6 метров за одну секунду. Почти семнадцать метров, которые неминуемо будут добавляться к длине тормозного пути и которые редко учитываются большинством водителей. Вот именно поэтому необходимо серьёзно отнестись хотя бы к теоретическому вычислению длины тормозного пути.

Что нужно для расчёта тормозного пути

Чтобы вычислить тормозной путь формула которого указана на рисунке с пояснениями, мало знать моментальные сухие данные.

Теоретически, для оценки тормозных характеристик машины необходимо использовать массу данных:

  • длину тормозного пути;
  • минимальное время, за которое тормозная система сработает;
  • диапазон изменения тoрмозных усилий;
  • алгоритм изменения тoрмозных усилий;
  • производительность тормозов в зависимoсти от нагрева;
  • качество дорожного покрытия;
  • эффективность подвески автомобиля;
  • степень износа и тип покрышек.

Здесь нужно учитывать целый ряд моментов. К примеру, эффективность работы тормозной системы в каждом автомобиле может быть разной и это само собой разумеется. Гидравлическая система тормозов даёт задержку минимум 0,2-03 с, а пневматика, установленная на большинстве грузовиков и автобусов и того больше, до 0,6 с. Кроме этого, есть такое понятие, как нарастание тормозного усилия с нуля до максимального значения и это также отбирает от 0,4 до 0,6 с, при этом влияние скорости движения на длину тормозного пути в этом случае увеличивается в квадрате, то есть при увеличении скорости в два раза, тормозной путь будет вчетверо длиннее.

Дополнительные составляющие тормозного пути

При вычислении эффективности тормозов очень большое значение имеет характеристика подвески и состояние шин. При чем тут подвеска? Очень просто. У нас под колёсами довольно редко встречается идеально ровный асфальт, а именно подвеска, точнее, амортизаторы, рессоры, торсионы и пружины как раз и прижимают колеса к поверхности, делая торможение и управление максимально эффективным. Если амортизатор неисправен, колеса подпрыгивают на ухабах и о полном контакте с покрытием не может быть и речи.

Давайте к этому прибавим кoэффициент сцепления резины с дорoгой — здесь огромное значение имеет состояние дороги, тип покрышки (зима или лето), рисунок протектора, геометрия, износ прoтектора и качество резиноматериала. Тесты показали, что на одном и том же автомобиле, но с разными покрышками, длина тормозного пути может изменяться до трёх-пяти метров, а о качестве пoкрытия и говорить нечего. Попробуйте сравнить тoрможение на сухом асфальте и на льду.

Как видим, факторов, влияющих на тормозной путь, а тем более на остановочный, достаточно много, поэтому предельная концентрация внимания за рулём — это гарантия безопасной езды. Проверяйте тормоза вовремя, не говорите по телефону за рулём и пусть все ваши дороги будут добрыми!

Как быстро автомобиль ускоряется, наверное, знает большинство автовладельцев. Даже если вы не замеряли динамику разгона своей машины, вы наверняка смотрели заводские технические характеристики вашего авто, где обычно автопроизводитель указывает минимально возможное время разгона с 0-100 км/час. Но теперь вопрос: сколько времени нужно, чтобы остановить вашу машину? Вы знаете это? Уверены, что нет. Но, оказывается, рассчитать расстояние тормозного пути можно достаточно легко с помощью простой формулы. Мы расскажем вам, как это делается.


Нет такой вещи во Вселенной или материи, которая может мгновенно остановиться. Также и любой автомобиль, когда вы нажимаете педаль тормоза, не сразу может остановиться. Дело в том, что для того чтобы автомобиль или любой объект в нашем мире остановился, необходимо, чтобы он потерял энергию, которая его движет. В результате у любого автомобиля есть тормозной путь, который он проезжает с момента нажатия педали тормоза до момента полной остановки. Это и есть тормозное расстояние машины.

Но на самом деле тормозной путь любого авто зависит не только от его характеристик и тормозной системы, но и от реакции водителя при нажатии педали тормоза. Ведь для того чтобы принять решение о необходимости торможения и нажать педаль тормоза, требуется время, которое хоть и минимально, но достаточно, чтобы машина успела проехать немаленький путь. Особенно это важно при большой скорости движения, где за какие-то доли секунды автомобиль проезжает приличное расстояние. Итак, в итоге, чтобы рассчитать реальную длину тормозного пути, нужно учитывать не только время и расстояние, пройденное автомобилем с момента нажатия водителем педали тормоза до момента остановки машины, но и время, необходимое для принятия решения о торможении. Дело в том, что при принятии решения о торможении мы тратим драгоценные секунды. Вот пример:

  • Время отклика : Прежде чем водитель нажмет педаль тормоза, он должен оценить дорожную ситуацию и определить, необходимо ли торможение. Также нужно понять, какое необходимо торможение – полная остановка автомобиля или простое снижение скорости. Обычно, согласно многочисленным исследованиям, большинству водителей для этого требуется около 0,1 секунды.
  • Время, необходимое для нажатия педали тормоза : После того, как водитель понял, что должен тормозить, необходимо еще примерно 0,8 секунды, для того чтобы переместить ногу с педали газа на педаль тормоза и нажать ее.

Кроме того, даже при нажатии педали тормоза есть еще небольшая потеря времени, связанная с тем, что при нажатии педали тормоза автомобиль, как правило, не начинает резко тормозить. А для того чтобы машина реально начала резко снижать скорость, надо усилить давление на педаль тормоза (пороговое время, необходимое для требуемого тормозного давления в тормозной системе). Также у всех автомобилей разное время отклика на нажатую педаль тормоза. Здесь все, конечно, зависит от конструкции тормозной системы и наличия различной электроники, контролирующей тормоза автомобиля.

Вы не поверите, но для того чтобы машина реально начала тормозить после нажатия педали тормоза, необходима еще почти 1 секунда времени. Вы представляете, как это много при движении на большой скорости? За эту лишнюю секунду вы можете проехать очень большой путь.

Что такое формула тормозного пути?

В общем, торможение автомобиля делится на два вида. Например, есть нормальное торможение, а есть экстренное, когда вам нужно резко остановить машину, чтобы избежать аварии.

При торможении в повседневной жизни, допустим, если вы хотите остановить автомобиль на светофоре, вы обычно нажимаете педаль тормоза намного плавнее и мягче, чем при необходимости полностью остановить автомобиль на парковке во дворе. В этом случае вы не применяете в машине максимальное тормозное усилие. При таком плавном и мягком торможении, как правило, тормозной путь (тормозное расстояние) увеличивается. Примерное расстояние тормозного пути при нормальном торможении можно рассчитать по следующей простой формуле:

(Скорость в км/ч: 10) x (скорость в км/ч: 10) = тормозной путь в метрах

При экстренном торможении педаль тормоза, как правило, нажата целиком и с полной силой. Из-за более высокой силы торможения обычно тормозной путь машины сокращается примерно в 2 раза. Поэтому длину тормозного пути можно также вычислить по следующей формуле:

(Скорость в км/ч: 10) x (скорость в км/ч: 10) / 2 = тормозной путь в метрах

Внимание: Вычисляемый по этим формулам тормозной путь является лишь приблизительным значением и подсказкой для водителей. На самом деле в реальности тормозной путь может быть как меньше, так и больше. Ведь расстояние тормозного пути зависит от навыков и опыта вождения водителя, от технической исправности автомобиля, его конструкции, марки, модели, состояния дорог, состояния протектора резины и многих других факторов, которые напрямую влияют на длину тормозного пути. Но благодаря этим формулам вы примерно сможете высчитать среднюю длину тормозного пути машины при определенной скорости движения. Это позволит вам скорректировать ваш стиль управления автомобилем, а также станет хорошим пособием для водителей-новичков.

Как рассчитать полное время остановки и итоговый тормозной путь?


Как мы уже сказали, чтобы рассчитать весь тормозной путь, нужно учитывать потерю времени при принятии водителем решения о торможении (то есть время реакции водителя). Для этого нужно использовать другую формулу, которая обеспечивает более точный приблизительный расчет тормозного расстояния, которое проедет автомобиль в момент принятия решения о необходимости остановки. Вот эта формула:

(Скорость в км/ч: 10) x 3 = путь реакции в метрах

В итоге, сделав вычисление по вышеуказанным формулам, вы можете вычислить приблизительный итоговый тормозной путь вашего автомобиля при любой скорости движения. Вот пример. Если вы управляете своим автомобилем со скоростью 50 км/ч, то с помощью приведенных формул вычислите следующие значения:

  • Тормозной путь при принятии решения о торможении на этой скорости (реакция на дорожную ситуацию + принятие решения о торможении + время, необходимое для перемещения ноги с педали газа на педаль тормоза, а также время отклика тормозной системы на нажатую педаль тормоза) составит где-то (50/10) х 3 = 15 метров. То есть пока вы будете принимать решение о торможении при скорости в 50 км/ч, ваша машина проедет 15 метров.
  • Тормозной путь при нормальном торможении (с момента нажатия педали тормоза до момента остановки машины) составит около (50/10) х (50/10) = 25 метров.
  • При экстренном торможении тормозной путь, как мы уже отметили, сокращается примерно в два раза. Соответственно, расчет тормозного расстояния автомобиля, который движется со скоростью 50 км/ч, будет выглядеть следующим образом: (50/10) x (50/10) / 2 = 12,5 метров.
  • В результате теперь мы можем вычислить реальный итоговый тормозной путь автомобиля. Так, при нормальном (не резком, а обычном) торможении итоговый тормозной путь составит около 40 метров. При экстренном торможении – не менее 28 метров.

Примечание: Обратите внимание, что если скорость автомобиля будет выше всего в два раза, его итоговый тормозной путь увеличится в четыре раза!!!

То есть мнение о том, что при увеличении скорости автомобиля в два раза тормозной путь увеличивается только в два раза, – это чистый воды миф среди многих автолюбителей. Так что имейте это в виду, когда садитесь за руль. Самое удивительное, что об этом не знают даже многие опытные водители.

Пример расчета тормозных и остановочных расстояний

Скорость, в км / ч

Путь, пройденный автомобилем

во время реакции водителя, в метрах

Тормозное расстояние, в метрах

(с момента нажатия педали тормоза

до полной остановки машины)

Итоговый тормозной путь, в метрах

6,25

13,75

Какие факторы влияют на торможение и тормозной путь?


Решающим значением для длины тормозного пути, конечно же, является скорость автомобиля, с которой он движется по дороге. Также на тормозной путь влияет качество установленной на машину тормозной системы. В том числе важную роль, несомненно, играет и состояние дороги (снег, лед, качество асфальта/бетона, трещины в дорожном покрытии, листья, лужи и т. п.). И само собой, не стоит забывать о состоянии шин автомобиля. Ведь в определенных случаях изношенная резина сильно увеличит тормозной путь автомобиля, так как не сможет передавать нормальную тормозную способность дорожному покрытию в отличие от новых шин, имеющих нормальное сцепление с дорогой.

Также ясно, что на мокрой поверхности тормозное расстояние машины больше, чем на сухом асфальте.

Не стоит забывать и об уровне подготовки водителя. Особенно важна, как мы узнали, для итогового тормозного пути скорость реакции водителя на дорожную ситуацию, требующую остановки автомобиля. Но скорость реакции за рулем зависит не только от опыта вождения. Например, знаете ли вы, что когда вы садитесь за руль в сонном состоянии (не выспались, устали или долго находились за рулем), то скорость реакции может замедлиться почти в два раза по сравнению со скоростью реакции хорошо отдохнувшего водителя.


В целом же на скорость принятия решения за рулем (скорость реакции) влияет много факторов: возраст водителя, алкогольное или похмельное состояние, употребление определенных медикаментов и в целом состояние здоровья. Так, при многих хронических заболеваниях скорость реакции многих водителей существенно снижается. Следовательно, все эти факторы серьезно влияют на тормозной путь автомобиля.

То же самое касается и отвлечения внимания из-за смартфонов, которыми так любят пользоваться за рулем многие водители, несмотря на строгий запрет согласно нашему действующему законодательству.

Как мы уже сказали, на тормозной путь также влияет время отклика тормозной системы автомобиля на нажатую педаль тормоза. Особенно это касается старых автомобилей. Современные же, как правило, оснащены уже новым поколением тормозов, которые мгновенно активируются за счет максимального тормозного давления, как только вы резко ударите ногой по педали тормоза (например, при экстренном торможении). Эта технология позволила существенно сократить итоговый тормозной путь современных машин.

Как повысить безопасность при управлении автомобилем?


Не зря основное правило вождения гласит о том, что водитель должен держать на дороге достаточную дистанцию до других автомобилей, чтобы оставалось пространство для экстренного торможения и для того, чтобы не спровоцировать ДТП. Но, с другой стороны, вы не должны держать дистанцию между автомобилями слишком большой. Помните, что все должно быть в меру. Вот некоторые правила вождения от экспертов:

  • В городском движении : Держите расстояние до других автомобилей около 15 метров.
  • На автомагистралях, шоссе и проселочных дорогах : При скорости движения около 100 км/ч держите дистанцию примерно 50 метров. При плохой видимости или на скользкой дороге дистанция до других машин должна быть увеличена в два раза. Например, при скорости в 100 км/ч на скользкой дороге держите расстояние до впереди идущей машины минимум в 100 метров.

Информационное издание: Новости гаи, дтп, штрафы пдд, ГИБДД, Экзамен ПДД онлайн. Техосмотр

Каждый водитель хоть раз да оказывался буквально в паре секунды от аварии, когда жизненно необходимо успеть затормозить. Однако встать, как вкопанный по команде автомобиль не может. Расстояние, которое он проедет с момента начала торможения до полной остановки и называют тормозным путём. Уметь прикинуть тормозной путь нужно, чтобы он всегда был меньше, чем расстояние до оказавшейся на пути помехи.

Длина пути торможения зависит от множества разных факторов. Тут и реакция водителя, и уровень работы тормозной системы автомобиля, и внешние факторы, вроде материала трассы и погодных условий. Ну и конечно, решающую роль играет скорость машины на момент торможения. Появляется вопрос — как рассчитать тормозной путь автомобиля при всех этих условиях? Для общих расчётов достаточно трёх главных факторов — тормозного коэффициента (Кэ), скорости движения (V) и коэффициента сцепления (Фс) с трассой.

Формула для расчёта тормозного пути автомобиля

Формула из таблицы, вычисляющая длину тормозного пути, выглядит так: S=Кэ*V*V/(254*Фс) . Тормозной коэффициент у обычного легкого автомобиля равняется единице. Коэффициент сцепления на сухой поверхности будет равен 0,7. Для примера, возьмём случай, когда машина движется по сухой трассе со скоростью в 60 км/ч. Тогда длина тормозного пути будет равна 1*60*60/(254*0,7)=20,25 метра. На льду же (Фс=0,1) торможение продлится в семь раз дольше — 141,7 метра!

По результату видим, как сильно длина тормозного пути автомобиля из таблицы зависит от состояния трассы и погодных условий.

Длина тормозного пути обратно пропорциональна коэффициенту сцепления с трассой. Проще говоря — чем хуже “держит” дорога, тем дольше машина тормозит. Посмотрим на изменения коэффициента (Фс) подробнее:

  • при сухом асфальте — 0,7;
  • на мокром асфальте — 0,4;
  • если укатан снег — 0,2;
  • обледеневшая дорога — 0,1.

Эти цифры позволяют нам увидеть, как изменится тормозной путь в зависимости от условий. Как уже говорилось, при скорости 60 км/ч на сухой дороге автомобиль будет тормозить 20,25 метра, а на льду — 141,7. На мокрой трассе дистанция торможения составит 35,4 метра, а на заснеженной — 70,8.

Типы торможений

Типы торможения

Стоит также учитывать, что большую роль играет способ торможения:

  1. Резкое нажатие может отправить автомобиль в неконтролируемый занос.
  2. Постепенное нажатие на педаль сработает при хорошей видимости и запасе времени, но его не применить в экстренной ситуации.
  3. Прерывистое торможение с несколькими нажатием на педаль до упора позволит быстро остановить машину, но также чревато потерей контроля.
  4. Ступенчатое нажатие позволит блокировать колёса, не потеряв контакт с педалью.

Торможение с ABS

Система ABS работает как раз по принципу ступенчатого торможения, а её основная задача — не отпустить машину в неконтролируемый занос. ABS не блокирует колёса полностью, тем самым оставляя водителю контроль над движением автомобиля. Обильные проверки показали, что ABS сократит тормозной путь на сухом или мокром асфальте, а также отлично работает на гравии. А вот в других условиях система частично теряет свою ценность.

В зимних условиях ABS увеличит тормозной путь на 15-30 метров при движении по снегу или льду. При этом система оставит водителю контроль над машиной, что может быть критически важно при движении по гололёду.

Таблица трения при разных скоростях

Помните, слабых места ABS — раскисшая земля и глина. На них тормозной путь также может стать дольше, чем при полностью “ручном” торможении. Но и контроль над машиной также останется.

Как определить скорость автомобиля по тормозному пути?

В тех случаях, когда затормозить вовремя всё же не удалось, необходимо определить, с какой скоростью двигался транспорт на момент начала торможения. Общая формула вычисления “стартовой” скорости торможения выглядит так — V = 0,5*t3*j + √2*S*j . В данном случае, роль играют следующие факторы:

  • — время нарастания замедления машины. Измеряется в секундах;
  • j — замедление автомобиля при торможении. Измеряется в м/с2. По ГОСТу на сухой трассе j=6,8 м ;
  • с2 , а на мокрой — 5 м/с2;
  • S — длина тормозного следа.

Возьмём условия, в которых tЗ=0,3 секунды, тормозной след 20 метров, а трасса сухая. Тогда скорость равна 0,5*0,3*6,8 + √2*20*6,8 = 1,02 + 19,22 = 20,24 м/с = 72,86 км/ч.

В основном для определения скорости в начале торможения используются три способа:

  1. Определение по тормозному пути.
  2. Определение по закону сохранения количества движения.
  3. Определение по деформации автомобиля.

Преимущества первого метода — простота и скорость, большое количество исследований, точный результат. Второй метод хорош тем, что его можно использовать при отсутствии следов торможения, он даёт точный результат и полезен при столкновении с неподвижными машинами. Третий отличается тем, что учитывает энергозатраты на деформацию машины.

Минусы у каждого метода также свои. В первом случае это невозможность использования при отсутствии следов шин. Во втором — громоздкие вычисления, а в третьем — большие объёмы того, что нужно учесть, и невысокая точность вычислений.

У какого автомобиля больше тормозной путь — у груженого под завязку или у пустого?
Больше половины людей ответят, что у груженого.
А на как обстоят дела на самом деле?

Для начала придется окунуться в «школьные годы чудесные», а именно — в физику за 6-й класс. Раздел «Силы трения». Окунаться будем не глубоко, по щиколотку.
Итак, смотрим на картинку. Перед нами — одноглазый Билли Бонс за рулем Фольксвагена. Он что-то увидел на дороге и вовсю тормозит. С точки зрения физики, и Фольксваген, и Билли Бонс — все это вместе называется «тело». На это тело действуют силы. Это сила тяжести, которая прижимает тело к земле mg , сила реакции опоры N , которая ей противодействует. Эти силы в простейшем случае, на горизонтальной поверхности, равны и направлены в разные стороны, а их равнодействующая равна нулю. Кроме них на движущееся тело действует еще одна сила — сила трения Fтр . Сила трения зависит от силы реакции опоры и коэффициента трения, она прямо пропорциональна им. А если точнее, равна просто их произведению: F тр. = μN .
Но сила реакции опоры равна массе тела, умноженной на ускорение свободного падения g: N = mg .
Подставим значение N в формулу силы трения:
F тр. = μmg

Поскольку на всей планете Земля ускорения свободного падения одинаковое, то делаем вывод, что сила трения зависит от коэффициента трения и массы тела, и больше ни от чего.

Если на дело действует какая-то сила, оно начинает ускоряться (напомним, что с точки зрения физики торможение — тоже ускорение, только с обратным знаком). Согласно второму закону Ньютона, это сила равна произведению массы на ускорение: F = ma
Значит, ускорение равно a = F / m .
На наше тело действует единственная сила — сила трения (равнодействующая остальных равна нулю, значит, они не оказывают влияния). Значит,
a = F тр. /m , то есть ускорение (замедление торможения) равно силе трения, деленной на массу Билли Бонса и его Фольксвагена.
Но сила трения равна F тр. = μmg . Подставим это значение в нашу формулу:
а = μmg/m . Масса, деленная на эту же массу, сокращается. Значит, а = μg
Итак, ускорение (в нашем случае — это интенсивность торможения) зависит только от коэффициента трения! Какая бы ни была масса тела, она у нас сокращается, то есть чем больше масса, тем больше будет и сила трения, причем точно на эту же самую величину.

Вроде бы уже все ясно. Но нам надо решить задачу до конца и вычислить тормозной путь. Это просто. Ускорение а равно скорости V , деленной на время t
a = V / t
Тогда
t = V / a = V / μg

Согласно Закону равноускоренного движения, расстояние S равно:
S = at 2 / 2
Тогда
S = μg (V / μg) 2 / 2 = (V 2 / μg) / 2 = V 2 / 2μg

Итак,



Тормозной путь зависит только от скорости и коэффициента трения, и не зависит от массы автомобиля.

Ну а поскольку ускорение свободного падения — величина постоянная, и равна 9.81 м/с 2 , то упрощенно можно считать так:
S = V 2 / 20μ

Так гласят незыблимые законы физики. Но если заглянуть в характеристики автомобилей, легко обнаружить, что у грузовиков тормозной путь больше, чем у легковушек. Выходит, они нарушают эти самые незыблимые законы? Конечно, нет. Для того, чтобы разобраться в этом, придется выйти далеко за пределы элементарной физики и детально знакомиться со свойствами тормозных систем (в частности, в разнице работы между «легковой» гидравлической и «грузовой» пневматической — а они разные), а также — в работе шины. В частности, в зависимости коэффициента трения шины от ее температуры, и, самое главное, от того, в какой момент начнется плавление резины. Чем раньше шина начнет плавиться — тем больше будет тормозной путь. А раньше начнет плавиться та шина, которая сильнее прижимается к асфальту. То есть — шина грузовика.
Тем не менее, в самом общем случае, когда скорости разумные, тормозной путь конкретного автомобиля не будет зависеть от того, насколько он нагружен. Не верьте тем людям, которые утверждают, что у сильно загруженного автомобиля он больше. Он такой же точно, как у пустого.

Что же касается автомобиля с прицепом, не оборудованным тормозами, то путем нехитрых преобразований мы получим такую формулу ускорения:
а = μg (1 + m пр. / m авт.)
Из чего видно, что сама масса прицепа не имеет значения, а важно только отношение массы прицепа к массе автомобиля: чем оно больше — тем больше ускорение и, стало быть, тормозной путь. Прямо пропорционально отношению масс автомобиля, который тормозит и прицепа, который тормозить не может. S = V 2 / 2μg(1 + (m пр. / m авт.))
Видно, что если масса прицепа будет равна половине массы автомобиля, то тормозной путь увеличится наполовину, то есть станет в полтора раза длиннее. А если масса прицепа равна массе автомобиля — то в два раза.

Статья написана по материалам лекций

А также дорожных условий. К примеру, при скорости движения 50км/ч величина среднего тормозного пути составит приблизительно 15 м, а при 100 км/ч – 60 м.

Учтите, что тормозной путь автомобиля зависит от множества факторов, таких как: скорость движения, вес автомобиля, дорожное покрытие, погодные условия, способ торможения, а также состояние колес автомобиля и его тормозной системы.

Определяйте тормозной путь автомобиля по следующей формуле: S = Kэ x V x V/(254 x Фc), где
S – тормозной путь автомобиля в ,
Кэ – тормозной коэффициент, который равен 1 у ,
V – скорость автомобиля (в км/ч) в начале торможения,
Фc – коэффициент с дорогой (разные показатели в зависимости от погодных условий),
0.7 – сухой асфальт,
0.4 – мокрая дорога,
0.2 – укатанный снег,
0.1 – обледенелая дорога.

Обратите внимание, что существует несколько различных способов торможения, а именно: плавное, резкое, ступенчатое и прерывистое. Плавное торможение применяйте в спокойной обстановке. Выполняйте постепенное увеличение давления на педаль тормоза, и это обеспечит плавное снижение скорости автомобиля. Именно при таком способе торможения вы получите самый большой тормозной путь .

Помните, что резкое торможение, когда вы сильно нажимаете на педаль тормоза, обычно приводит к колес, а значит и к управления и заносу автомобиля. Если вы выбираете ступенчатое торможение, то несколько раз нажимайте на педаль, но каждое последующее нажатие делайте с большим усилием, и так до полной остановки автомобиля.При прерывистом сильно нажимайте на педаль, почти до момента колес, а затем отпускайте педаль. Следуйте такому же принципу до того, как автомобиль полностью не остановится.

Любой автомобилист должен уметь определить тормозной путь . От этого порой зависит безопасность водителя и тех, кто находится в машине помимо него. Что такое тормозной путь и как его определить, чтобы избежать неприятностей на дороге?

Вам понадобится

  • автомобиль, дорога

Инструкция

Тормозной путь – это расстояние, которое проходит после срабатывания тормозной системы и до окончательной остановки. Величина тормозного пути зависит от множества факторов: скорости движения автомобиля, способа торможения и условий, задающихся дорогой. Чем больше скорость, тем больше и тормозной путь .

Помимо обозначенных факторов наиболее важны дорожное покрытие и его состояние, масса автомобиля, а также технические характеристики и исправность колес и тормозной системы. Наименьшей длина тормозного пути будет на сухой заасфальтированной , наибольшей – на льду. Соответственно, вместе с увеличением тормозного пути увеличивается .

Конечно, в экстренной ситуации рассчитать тормозной путь крайне трудно, но следует представлять возможности своего автомобиля в этом плане, чтобы сориентироваться в нужный момент. Существует , по которой можно определить тормозной путь . Автомобилистам рекомендуется воспользоваться ею прежде, чем садиться за руль, так как это может предотвратить многие нежелательные инциденты.

Формула выглядит так: S = Kэ x V x V/(254 x Фc). Необходимо пояснить в ней условные обозначения. S – это длина тормозного пути в метрах, Кэ – коэффициент торможения, который для легковых автомобилей всегда равен единице, V – начальная скорость при торможении, измеряющаяся в км/ч, а Фс – коэффициент сцепления с дорогой, зависящий от ее состояния (при сухом асфальте – 0,7, при мокрой дороге – 0,4, в случае укатанного – 0,2, и 0,1, если покрыта льдом). Определение тормозного пути – простое и полезное действие, доступное каждому автомобилисту. Достаточно подставить в формулу цифры, соответствующие конкретной ситуации и параметрам вашей .

Источники:

  • Как рассчитать тормозной путь в 2019

Правильно подобранные автомобильные шины обеспечивают безопасность и реализацию технических характеристик автомобиля. Если установить шины, рекомендуемые автопроизводителем, нет возможности, необходимо подобрать взаимозаменяемый типоразмер покрышек, учитывая несколько параметров.

Инструкция

Расшифруйте типоразмер автомобильных шин. Например, в обозначении 175/65R14, первые 3 цифры означают ширину шины в миллиметрах, 4 и 5 цифры – профиль шины (в % от ширины). Буква R является маркировкой радиальной шины, а последние 2 цифры обозначают диаметр диска в дюймах.

Подберите шины, незначительно отличающиеся по физическим размерам от «родных». Допустимым считается отклонение по диаметру до 5% и по ширине до 20%. Сравните полный диаметр колеса , установленного на автомобиль, с новым типоразмером. Общий объем = диаметр диска х 25,4 мм. + (ширина шины х (профиль шины/100) х 2). Например, если вы хотите сменить типоразмер 175/65R14 на 195/50R15, подставьте в эту формулу соответствующие параметры из этих обозначений. Получив в результате вычислений два значения (583,1 мм. и 576 мм.), найдите их разность и процент, который она составляет от первоначального объема. В данном примере разница 7,1 мм. составляет 1,2% от первоначального объема 583,1 мм. и является допустимым значением. Следовательно, вы можете без опасений сменить типоразмер автомобильных шин.

Сравните высоту профиля и ширину шины. Например, вы хотите сменить размер 175/65R14 на 185/60R14. Умножьте ширину шины на профиль (175 мм. х 0,65 и 185 мм. х 0,60). Разность полученных значений (113,75 мм. – 110мм. = 2,75мм.) составляет 2,4% от исходной высоты профиля (113,75 мм.). Разница в ширине шин (185 мм. -175 мм.) составляет 10мм. и 5,7% от ширины 175 мм. Таким образом, можно сделать вывод о том, что типоразмеры 175/65R14 и 185/60R14 являются взаимозаменяемыми.

Полезный совет

Просмотрите руководство по эксплуатации вашего автомобиля и найдите в нем информацию относительно рекомендуемых типоразмеров покрышек. Помните о том, что летние и зимние шины могут отличаться по размеру. Типоразмер также указывается на боковине покрышки.

Тормозной путь — это важная техническая характеристика автомобиля. Однако она зависит не только от работы тормозной системы, но и от множества других факторов, например, типа шин, установленных на автомобиль.

Тормозной путь

Тормозной путь представляет собой величину расстояния, которое рассматриваемый автомобиль успел проехать с момента срабатывания тормозной системы до момента полной остановки транспортного средства.2 / (254 * Фс). В указанной формуле символом S обозначается длина тормозного пути, выраженная в метрах, а символом v — скорость движения. выраженная в километрах в час. В свою очередь, обозначение Кэ указывает величину тормозного коэффициента, который для легкового автомобиля равен 1, а символ Фс — коэффициент сцепления с дорогой.

Таким образом, наиболее сложным для определения в этой формуле является значение коэффициента Фс. Обыкновенно в качестве его значения используются следующие цифры: коэффициент принимается равным 0,7 в случае движения на резине без шипов по сухому асфальту по ровной траектории, 0,4 — при движении в тех же условиях по мокрой дороге, 0,2 — при движении по укатанному снегу и 0,1 — при движении по обледенелой дороге.

Однако следует принимать во внимание, что расчет длины тормозного пути по этой формуле является приблизительным, поскольку учитывает только основные факторы — скорость и погодные условия. Вместе с тем, влияние на длину тормозного пути оказывают дополнительные факторы, такие как характер дорожного покрытия или тип покрышек, установленных на конкретном транспортном средстве. Свое воздействие оказывают также способ торможения, используемый водителем в рассматриваемом случае, и другие причины. Они могут оказывать весьма существенное влияние на длину тормозного пути, будучи способными изменить ее в несколько раз.

Источники:

  • Расчет длины тормозного пути автомобиля
  • Как рассчитать тормозной путь

Реакция, тормозной путь и формула

от Zutobi · Обновлено 14 июня 2021 г.

Важной частью предотвращения наезда сзади является знание вашего тормозного пути и того, как быстро ваш автомобиль может полностью остановиться. Создайте запас безопасности, обеспечивающий достаточное расстояние между идущим впереди транспортным средством, чтобы было время среагировать и безопасно остановиться. Чтобы полностью остановиться до столкновения, необходимо мысленно рассчитать тормозной путь.

Знание того, как скорость влияет на тормозной путь, жизненно важно для безопасного водителя. Но сначала давайте рассмотрим важную информацию.

Что такое расстояние реакции?

Расстояние реакции — это расстояние, которое проходит ваш автомобиль, пока вы реагируете. Таким образом, время, которое проходит с момента появления опасности до фактического начала торможения. Время реакции варьируется от водителя к водителю.

Что такое тормозной путь?

Тормозной путь — это расстояние, которое требуется вашему автомобилю для полной остановки, начиная с момента, когда вы нажимаете на тормоз.

Что такое тормозной путь?

Тормозной путь = путь реакции + тормозной путь. Таким образом, расстояние, необходимое вашему автомобилю для полной остановки с момента обнаружения опасности.

Формула тормозного пути

Скорость очень сильно влияет на вашу способность вовремя останавливаться и существенно влияет на ваши шансы попасть в аварию:

  1. При 30 милях в час вам нужно примерно 120 футов, чтобы добраться до полная остановка (65 футов для реакции и 55 футов для торможения) в хороших условиях.
  2. На скорости 60 миль в час вам нужно примерно 360 футов для полной остановки (130 футов для реакции и 190 футов для торможения) в хороших условиях.

Небольшое увеличение скорости также влияет на тормозной путь . Увеличение скорости всего на 10 миль / ч с 50 до 60 миль / ч увеличивает общий тормозной путь на 40%.

Как правило, удвоение скорости увеличивает тормозной путь в четыре раза, а утроение скорости увеличивает тормозной путь в девять раз.

Другие факторы, влияющие на тормозной путь

Мокрые и другие скользкие условия, а также сухие грунтовые дороги также добавляют значительный тормозной путь к полному тормозу. На тормозной путь влияют:

  • время реакции
  • состояние шин, включая глубину протектора и давление воздуха
  • дорожные условия
  • погодные условия
  • состояние транспортного средства и тормозная способность
грунтовые дороги требуют более длинного тормозного пути, чтобы довести автомобиль до полная остановка

Как тренироваться тормозной путь

Знаете ли вы, что попадание на задний борт — одна из основных причин дорожно-транспортных происшествий в Великобритании? Знание тормозного пути вашего автомобиля в различных условиях является важной частью обеспечения безопасности на дорогах, и поэтому тормозной путь является частью британского экзамена по теории вождения, который вы должны сдать перед получением водительских прав.Однако после того, как экзамен будет завершен, о нем можно легко забыть.

Освежите свои знания о тормозном пути автомобиля с помощью нашего руководства по вычислению тормозного пути по формуле тормозного пути.

Что такое тормозной путь?

Тормозной путь — это расстояние, пройденное между вами, увидев опасность, и остановив автомобиль в ответ на нее. Он зависит от ряда переменных, поэтому он будет меняться в зависимости от скорости вашего движения, погодных и дорожных условий, а также внимательности водителя.Однако простое осознание этого поможет вам оставаться на правильном расстоянии и скорости.

Чем быстрее едет машина, тем труднее ее быстро остановить. Тормозной путь особенно важен в зонах со скоростью 30 миль в час вокруг школ, больниц и домов. Двигаясь по этим местам, спросите себя, сможете ли вы вовремя остановиться, если кто-то выйдет на улицу.

Тормозной путь = расстояние мысли + тормозной путь

Что такое расстояние мышления?

Расстояние мышления — это расстояние, которое вы путешествуете в автомобиле от точки обнаружения опасности до точки начала торможения или поворота.Время на обдумывание обычно составляет от 0,5 до 2 секунд, хотя оно может увеличиваться, если водитель устал, отвлечен или принимает лекарства. Чрезвычайно важно, чтобы водители всегда оставались бдительными и бдительными, чтобы минимизировать расстояние для размышлений.

Расстояние мыслей составляет примерно 1 фут на каждую милю в час, на которой вы едете, например, автомобиль, движущийся со скоростью 30 миль в час, проедет 30 футов до того, как будут задействованы тормоза.

Что такое тормозной путь?

Тормозной путь — это расстояние, которое вы пройдете от начала торможения до полной остановки автомобиля.Как известно, тормозной путь рассчитать сложно, потому что дорожные условия и сцепление с дорогой могут сильно повлиять на него. Однако вы можете использовать следующие таблицы в качестве примеров тормозного пути в средних условиях.

Как рассчитать тормозной путь

Умножьте скорость, с которой вы путешествуете, на 0,5, начиная с 2:

.

Чтобы преобразовать тормозной путь из футов в метры, умножьте расстояние на 3,3:

Джек Андервуд

Вам понравился этот пост в блоге? да | Нет
9 человек считают этот отзыв полезным

тормозной путь | Формула тормозного пути

Узнайте о тормозном пути, общем тормозном пути и плавных остановках.

Вернуться к темам испытаний на разрешение


Правильное торможение является важной частью безопасности водителя.

Изучение некоторых вещей об использовании тормозов сделает вас более безопасным водителем и поможет вам пройти тест на разрешение, чтобы получить разрешение на обучение во Флориде.

После того, как вы посмотрели видео и прочитали приведенные ниже руководства по торможению, тормозному пути и тому, как ваша скорость влияет на вашу способность останавливаться, мы рекомендуем вам пройти наш практический тест по торможению, чтобы определить, понимаете ли вы эту тему.

Пройдите тест на практику торможения


Темы торможения и тормозного пути

На ваш тормозной путь влияют два фактора: время восприятия и время реакции. Восприятие — это когда вы видите опасность, а Время реакции — это время, пока вы не нажмете педаль тормоза. Если вы отвлекаетесь, это добавляет дополнительное время к тормозному пути.


Чем быстрее вы едете, тем больше времени требуется для остановки.Это означает, что превышение скорости увеличивает тормозной путь и силу удара. Если вы удвоите скорость, то ваш тормозной путь и сила удара увеличатся в 4 раза.


Общий тормозной путь представляет собой комбинацию расстояния реакции, расстояния восприятия и расстояния торможения. Время восприятия и реакции добавляет 55 футов (всего 110 футов) к общему тормозному пути.


Нажатие на тормоза чрезвычайно опасно.Внезапные остановки обычно вызваны невниманием водителей и являются основной причиной наездов сзади. Узнайте, как делать плавные безопасные остановки.


Влияние времени восприятия и реакции на тормозной путь

На ваш тормозной путь влияют два фактора: время восприятия и время реакции.

Восприятие — это когда вы видите опасность и понимаете, что вам нужно остановиться, а реакция — это время, необходимое вам, чтобы нажать на тормоз. Каждый из этих двух факторов увеличивает задержку процесса торможения.

Ниже показано увеличение времени и расстояния при торможении, вызванное восприятием и реакцией на скорости 50 миль в час.

  • Время восприятия = от 3/4 секунды до 1 секунды.
  • Расстояние восприятия = 55 футов.
  • Время реакции = от 3/4 секунды до 1 секунды.
  • Расстояние реакции = 55 футов.

Расстояние восприятия и реакции вместе составляет 110 футов к общему тормозному пути — это не включает фактический тормозной путь.


Как скорость влияет на тормозной путь

Чем быстрее вы едете, тем больше времени требуется для остановки. Это означает, что превышение скорости увеличивает тормозной путь и силу удара.

Как скорость влияет на тормозной путь и удар

Удвойте скорость с 20 до 40 миль / ч ваш тормозной путь и сила удара в 4 раза больше.

Увеличьте скорость втрое с 20 до 60 миль в час, а тормозной путь и ударную нагрузку в 9 раз больше.

Увеличьте скорость в четыре раза с 20 до 80 миль в час, и ваш тормозной путь и ударная нагрузка увеличатся в 16 раз.

Увеличение тормозного пути и силы удара — одна из причин, по которой превышение скорости настолько опасно.


Общий тормозной путь

Общий тормозной путь не так прост, как время, необходимое вашей машине, чтобы остановиться после нажатия на тормоз.

На скорости 50 миль в час ваш общий тормозной путь составляет не менее 268 футов.

268 футов — это комбинация:

  • 55 футов для восприятия.
  • 55 футов для реакции.
  • 158 футов для торможения.

Тормозной путь основан на идеальных условиях с исправными тормозами. Например, если идет дождь или темно, общий тормозной путь увеличится.


Торможение — обеспечение плавных остановок

Выполнение плавных остановок, а не нажатие на тормоза, важно, потому что это поможет избежать наезда сзади и держать автомобиль под контролем при повороте.

Вот шаги, которые необходимо выполнить для плавной и безопасной остановки:

  1. Перед остановкой проверьте зеркала и слепые зоны.
  2. Уберите ногу с педали газа, и ваша машина начнет тормозить.
  3. Нажмите педаль тормоза, чтобы включить стоп-сигналы.
  4. Плавно нажимайте на педаль тормоза.

Плавные остановки — хорошая привычка, которая поможет вам избежать столкновения с автомобилем позади вас. Плавные остановки также уменьшают износ тормозов.

Уход за водителем — знай свой тормозной путь

What a o ne-s econd a dvantage c an m ean t 905 9249

Каждый хочет подарить больше времени, но насколько ценным может быть для вас всего одна жалкая, убогая секунда? Когда дело доходит до использования тормозов на шоссе, это может означать … ну, намного больше, чем вы можете себе представить.

Исследования показали, что среднему водителю требуется от половины до трех четвертей секунды, чтобы ощутить необходимость нажать на тормоз, и , еще , три четверти секунды, чтобы переместить ногу с газа на тормоз. педаль. Время реакции у всех разное, но это может составлять полторы секунды между моментом, когда вы впервые начинаете понимать, что у вас проблемы, и даже до того, как вы начинаете замедляться.

Это фундаментально — физиология человека не меняется.Но давайте посмотрим, как это влияет на вашу способность останавливать машину.

В таблице ниже показано расстояние, которое требуется среднему автомобилю, чтобы остановиться на сухом асфальте с разной скоростью, включая расстояние, пройденное всего за одну секунду времени восприятия и реакции.

Скорость Расстояние восприятия / реакции Тормозной путь Общий тормозной путь Примерно равно количеству автомобилей длиной (на 15 футов)
30 миль / ч 44 фута 45 футов 89 футов 6
40 миль / ч 59 футов 80 футов 139 футов 9
50 миль / ч 73 футов 125 футов 198 футов 14
60 миль / ч 88 футов 180 футов 268 футов 18
70 миль / ч103 футов 245 футов 348 футов 23
80 миль / ч 117 футов 320 футов 439 футов 29

Обратите внимание, что когда вы удваиваете скорость — скажем, с 30 миль в час до 60 или с 40 до 80 — ваш общий тормозной путь увеличивается более чем вдвое: он увеличивается в три раза!

Когда дело доходит до торможения, всегда следуйте этим трем ключевым принципам безопасного вождения:

  • Снизьте скорость. Чем медленнее вы едете, тем короче ваш тормозной путь.
  • Смотрите далеко вперед, чтобы увеличить время предупреждения. Всегда глядя как можно дальше по дороге, вы быстрее увидите возникающие опасности и стоп-сигналы автомобилей, идущих впереди вас.
  • Рано двигайте ногой. Снимая ногу с педали газа и слегка нажимая на педаль тормоза при первом признаке того, что вам нужно снизить скорость, вы резко ускоряете время реакции и защищаете свою заднюю часть, давая водителям позади вас более ранний предупреждающий знак.

Классическое исследование, проведенное в 1980-х годах, показало, что 90 процентов всех аварий можно было бы избежать, если бы водитель среагировал всего на секунду раньше. Использование этих советов по безопасному торможению может дать вам необходимое преимущество в одну секунду.

Как рассчитать тормозной путь!

Во время теоретического теста вас спросят о тормозном пути при определенных скоростях, цифры перечислены ниже:

Теперь вы можете просто попытаться запомнить эти цифры, но их может быть довольно сложно запомнить, поэтому это методы для расчета цифр, вам просто нужно относительно хорошо разбираться в математике.


Дистанция мышления:

Это время, которое в среднем требуется, чтобы «подумать» о торможении в случае опасности. Дистанция мышления (в футах) — это просто скорость. Например, при 20 милях в час расстояние для мышления составляет 20 футов, при 30 милях в час — 30 футов и так далее.

Тормозной путь:

Это то, сколько в среднем требуется времени, чтобы «затормозить» до полной остановки при возникновении опасности, приведенный ниже расчет может решить это:

((Первая цифра скорости [в миль / ч] / 2) x Скорость)

Например, тормозной путь на скорости 40 миль в час:

Первая цифра скорости: 4

4, деленная на 2 = 2

2 X 40 = 80 футов

В кодексе автомобильных дорог говорится, что Фактический тормозной путь на скорости 40 миль в час составляет 78 футов, мы получили 80 футов.Таким образом, это 2 фута, однако на тесте по теории вам будут предложены вопросы с несколькими вариантами ответов, поэтому просто выберите ответ, наиболее близкий к тому, который вы рассчитали.

Общий тормозной путь:

Это просто «мысленное расстояние», добавленное к «тормозному пути», поэтому на скорости 40 миль в час это будет 40 + 80 = 120 футов. Опять же, мы находимся в двух футах от фактического ответа, но этих расчетов должно быть достаточно, чтобы проработать его для вашего теоретического теста.


Вышеуказанные книги являются незаменимой покупкой при обучении вождению.

В реальном мире

Как вы можете подозревать, цифры в Правилах дорожного движения не очень точны в реальном мире, это связано с различными факторами, которые могут повлиять на ваш тормозной путь, например, автомобиль (состояние / качество шин и тормозов, вес автомобиля) и водителя (отвлекаются ли они, например, пользуются мобильным телефоном или разговаривают с пассажирами? Устали / утомлены?).

Если мы возьмем пример водителя, который отвлекается на использование своего мобильного телефона при движении со скоростью 50 миль в час, и как это влияет на расстояние их мышления.Водитель смотрит на свой телефон 2 секунды, это недолго, верно? На скорости 50 миль в час вы путешествуете со скоростью 75 футов в секунду, так что за те 2 секунды, которые вы посмотрели в свой телефон, вы проехали 150 футов или 11,4 длины автомобиля. Другими словами, если бы перед вами было другое транспортное средство длиной в 11 машин, вы бы ударили его со скоростью 50 миль в час, прежде чем даже поставили ногу на тормоз или даже не оторвались от телефона!

Если у вашего автомобиля также были недопустимые шины с низким протектором, а тормозные колодки были сильно изношены, это также резко увеличило бы ваш тормозной путь.Отвлеченный водитель с автомобилем в плохом состоянии — это ожидающая авария авария, у него очень мало шансов среагировать в аварийной ситуации.

Что вы можете сделать?

  1. Не отвлекайтесь — например, Мобильный телефон, пассажиры, громкая музыка
  2. Поддерживайте свой автомобиль в хорошем состоянии — Выполняйте еженедельные проверки безопасности Show Me Tell Me и обслуживайте свой автомобиль в соответствии с руководством производителя (обычно один раз в год или каждые 10 000 миль)
  3. Соблюдайте безопасную дистанцию ​​ — Правила дорожного движения рекомендуют сохранять КАК МИНИМУМ 2 секунды от впереди идущего автомобиля (в хороших сухих дорожных условиях).Чтобы оценить 2-секундный промежуток, вы ждете, пока впереди идущая машина не пролетит неподвижный объект (например, фонарный столб или дорожный знак), а затем отсчитываете 2 секунды, это можно сделать, произнеся фразу «только дурак тормозит две секунды. правило »(на произнесение которого требуется 2 секунды), если вы можете произнести фразу до достижения фиксированного объекта, тогда вы находитесь на безопасном расстоянии позади, если вы не можете произнести фразу, они увеличивают ваше следующее расстояние (т.е. машина впереди). Во влажных условиях должен сохраняться, по крайней мере, 4-секундный интервал, а в условиях обледенения — как минимум 20-секундный интервал.

Готовы ли вы к внезапной остановке идущего впереди автомобиля?

Езда по двойной проезжей части со скоростью 70 миль в час, капот автомобиля впереди распахивается, заставляя его резко и резко тормозить

Транспортные средства впереди внезапно останавливаются

Автомобиль, идущий впереди, внезапно тормозит, может ли водитель сзади вовремя отреагировать?

Шина идущего впереди грузовика ударяет, можете ли вы вовремя остановиться, чтобы не врезаться в заднюю часть грузовика?

Онлайн-конвертеры единиц измерения

Случайный преобразователь

Онлайн-конвертеры единиц измерения

Конвертер длины и расстоянияПреобразователь массыКонвертер объема сухого воздуха и общих измерений при варкеПреобразователь площадиПреобразователь объёма и общего измерения при варкеПреобразователь температурыПреобразователь давления, напряжения, модуля ЮнгаПреобразователь энергии и работыПреобразователь силыКонвертер силыКонвертер времениЛинейный конвертер скорости и скоростиКонвертер углового расходаПреобразователь топливной эффективности, расхода топлива и информации о расходе топливаКонвертер единиц Хранение данныхКурсы обмена валютЖенская одежда и размеры обувиМужская одежда и размеры обувиКонвертер угловой скорости и частоты вращенияКонвертер ускоренияКонвертер углового ускоренияКонвертер плотностиКонвертер удельного объемаПреобразователь момента инерцииПреобразователь момента силыКонвертер крутящего моментаПреобразователь удельной энергии, теплоты сгорания (на единицу температуры) Преобразователь интерваловКонвертер коэффициента теплового расширенияПреобразователь теплового сопротивленияПреобразователь теплопроводности Конвертер удельной теплоемкости ter Конвертер скорости передачиКонвертер уровня звукаКонвертер чувствительности микрофонаКонвертер уровня звукового давления (SPL) Конвертер уровня звукового давления с выбираемым эталонным давлениемКонвертер яркостиКонвертер яркостиКонвертер яркостиКонвертер разрешения цифрового изображенияПреобразователь частоты и длины волныОптическая мощность (диоптрия) в преобразователь фокусного расстоянияПреобразователь оптической мощности (диоптрий) в увеличение (X) Конвертер электрического заряда Конвертер плотности зарядаКонвертер поверхностной плотности зарядаКонвертер объёмной плотности заряда Конвертер электрического токаЛинейный преобразователь плотности токаПреобразователь плотности поверхностного токаПреобразователь напряженности электрического поляПреобразователь электрического потенциала и напряженияПреобразователь электрического сопротивленияПреобразователь электрического сопротивленияПреобразователь электрической проводимостиПреобразователь электрической проводимостиПреобразователь емкостиПреобразователь индуктивностиПреобразователь реактивной мощности переменного токаПреобразователь единиц магнитного поля в ваттах и ​​дБм Конвертер плотности потока Конвертер мощности поглощенной дозы излучения, Конвертер мощности дозы полного ионизирующего излученияРадиоактивность.Преобразователь радиоактивного распада Преобразователь радиационного воздействияРадиация. Конвертер поглощенной дозы Конвертер метрических префиксов Конвертер передачи данных Конвертер единиц типографии и цифрового изображения Конвертер единиц измерения объема древесиныКалькулятор молярной массыПериодическая таблица

Этот онлайн-конвертер единиц измерения позволяет быстро и точно переводить многие единицы измерения из одной системы в другую. Страница преобразования единиц представляет собой решение для инженеров, переводчиков и для всех, чья деятельность требует работы с величинами, измеренными в различных единицах.

Вы можете использовать этот онлайн-конвертер для преобразования нескольких сотен единиц (включая метрическую, британскую и американскую) в 76 категорий или нескольких тысяч пар, включая ускорение, площадь, электрическую энергию, энергию, силу, длину, свет, массу, массовый расход, плотность, удельный объем, мощность, давление, напряжение, температура, время, крутящий момент, скорость, вязкость, объем и емкость, объемный расход и многое другое.
Примечание. Целые числа (числа без десятичной точки или показателя степени) считаются точными до 15 цифр, а максимальное количество цифр после десятичной точки равно 10.», То есть« умножить на десять в степени ». Электронная нотация обычно используется в калькуляторах, а также учеными, математиками и инженерами.

Преобразователи общих единиц

Конвертер длины и расстояния : метр, километр, сантиметр, миллиметр, нанометр, ярд, фут, дюйм, парсек, световой год, астрономическая единица, расстояние до Луны (от Земли до Луны), лига , миля, морская миля (международная), сажень, длина кабеля (международная), точка, пиксель, калибр, планковская длина…

Конвертер массы : грамм, килограмм, миллиграмм, тонна (метрическая), фунт, унция, камень (США), камень (Великобритания), карат, зерно, талант (библейский греческий), драхма (библейский греческий), денарий (библейский римский), шекель (библейский иврит), масса Планка, масса протона, атомная единица массы, масса электрона (покой), масса Земли, масса Солнца …

Сухой объем и стандартные измерения при приготовлении пищи : литр, бочка сухой (США), пинта сухой (США), квартовый сухой (США), peck (США), peck (Великобритания), bushel (США), bushel (UK), cor (библейский), homer (библейский), ephah (библейский) ), seah (библейский), omer (библейский), cab (библейский), log (библейский), кубометр.

Конвертер площади : миллиметр², сантиметр², метр², километр², гектар, акр, дюйм², фут², ярд², миля², сарай, круглый дюйм, поселок, роуд, стержень², окунь², усадьба, шест², сабин, арпент, куерда, квадратная верста, квадратный аршин, квадратный фут, квадратный сажень, площадь Планка …

Конвертер объёма и общих единиц измерения температуры : метр³, километр³, миллиметр³, литр, гектолитр, миллилитр, капля, бочка (масло), бочка (США) ), баррель (Великобритания), галлон (США), галлон (Великобритания), кварта (США), кварта (Великобритания), пинта (США), пинта (Великобритания), баррель (нефть), баррель (США), баррель (Великобритания ), галлон (США), галлон (Великобритания), кварта (США), кварта (Великобритания), пинта (США), пинта (Великобритания), ярд³, фут³, дюйм³, регистровая тонна, 100 кубических футов…

Преобразователь температуры : кельвин, градус Цельсия, градус Фаренгейта, градус Ренкина, градус Реомюра, температура Планка.

Преобразователь давления, напряжения, модуля Юнга : паскаль, килопаскаль, мегапаскаль, миллипаскаль, микропаскаль, нанопаскаль, атмосферно-техническая, стандартная атмосфера, тысячи фунтов на квадратный дюйм, фунт / кв. Дюйм, ньютон на метр², бар, миллибар, килограмм-сила / метр², грамм- сила / сантиметр², тонна-сила (короткая) / фут², фунт-сила / фут², миллиметр ртутного столба (0 ° C), дюйм ртутного столба (32 ° F), сантиметр водяного столба (4 ° C), фут водяного столба (4 ° C) , метр морской воды…

Конвертер энергии и работы : джоуль, килоджоуль, мегаджоуль, миллиджоуль, мегаэлектронвольт, электрон-вольт, эрг, киловатт-час, мегаватт-час, ньютон-метр, килокалория (IT), калория (пищевая), Британские тепловые единицы (IT), мегабтеки (IT), тонна-час (охлаждение), тонна нефтяного эквивалента, баррель нефтяного эквивалента (США), мегатонна, тонна (взрывчатые вещества), килограмм в тротиловом эквиваленте, дин-сантиметр, грамм-сила-сантиметр, килограмм-сила-метр, килопонд-метр, фут-фунт, дюйм-фунт, энергия Планка …

Преобразователь мощности : ватт, киловатт, мегаватт, милливатт, лошадиные силы, вольт-ампер, ньютон-метр / секунда, джоуль / секунда, мегаджоуль в секунду, килоджоуль в секунду, миллиджоуль в секунду, джоуль в час, килоджоуль в час, эрг в секунду, британские тепловые единицы (IT) в час, килокалории (IT) в час…

Преобразователь силы : ньютон, килоньютон, миллиньютон, дин, джоуль / метр, джоуль / сантиметр, грамм-сила, килограмм-сила, тонна-сила (короткая), кип-сила, килопунт-сила, фунт-сила сила, унция-сила, фунтал, фунт-фут в секунду², pond, sthene, грав-сила, миллиграв-сила …

Преобразователь времени : секунда, миллисекунда, наносекунда, пикосекунда, минута, час, день, неделя, месяц, год, декада, век, тысячелетие, планковское время, год (юлианский), год (високосный), год (тропический), год (сидерический), год (григорианский), две недели, встряска…

Конвертер линейной скорости и скорости : метр / секунда, километр / час, километр / секунда, миля / час, фут / секунда, миля / секунда, узел, узел (Великобритания), скорость света в вакууме, космический скорость — первая, космическая скорость — вторая, космическая скорость — третья, скорость Земли, скорость звука в чистой воде, Мах (стандарт СИ), Мах (20 ° C и 1 атм), ярд / секунду …

Угол Конвертер : градус, радиан, град, гон, минута, секунда, знак, мил, оборот, круг, поворот, квадрант, прямой угол, секстант.

Конвертер топливной экономичности, расхода топлива и экономии топлива : метр / литр, километр / литр, миля (США) / литр, морская миля / литр, морская миля / галлон (США), километр / галлон (США), литр / 100 км, галлон (США) / миля, галлон (США) / 100 миль, галлон (Великобритания) / миля, галлон (Великобритания) / 100 миль …

Конвертер чисел : двоичный, восьмеричный, десятичный, шестнадцатеричный, основание-3, основание-4, основание-5, основание-6, основание-7, основание-9, основание-10, основание-11, основание-12, основание-13, основание-14, основание-15, основание-20, основание-21, основание-22, основание-23, основание-24, основание-28, основание-30, основание-32, основание-34, основание-36…

Преобразователь единиц информации и хранения данных : бит, байт, слово, четверное слово, MAPM-слово, блок, килобит (10³ бит), кибибит, кибибайт, килобайт (10³ байтов), мегабайт (10⁶ байтов), гигабайт (10⁹ байтов), терабайт (10¹² байтов), петабайт (10¹⁵ байтов), эксабайт (10¹⁸ байтов), гибкий диск (3,5 ED), гибкий диск (5,25 HD), Zip 250, Jaz 2 ГБ, CD (74 минут), DVD (2 слоя 1 сторона), диск Blu-ray (однослойный), диск Blu-ray (двухслойный) …

Обменный курс валюты : евро, доллар США, канадский доллар, британский фунт стерлингов, японская иена, швейцарский франк, аргентинское песо, австралийский доллар, бразильский реал, болгарский лев, чилийское песо, китайский юань, чешская крона, датская крона, египетский фунт, венгерский форинт, исландская крона, индийская рупия, индонезийская рупия, новый израильский шекель , Иорданский динар, малайзийский ринггит, мексиканское песо, новозеландский доллар, норвежская крона, пакистанская рупия, филиппинское песо, румынский лей, российский рубль, саудовский риял, сингапурский доллар, Южноафриканский рэнд, южнокорейский вон, шведская крона, новый тайваньский доллар, тайский бат, турецкая лира, украинская гривна…

Размеры женской одежды и обуви : женские платья, костюмы и свитера, женская обувь, женские купальные костюмы, размер буквы, бюст, дюймы, естественная талия, дюймы, заниженная талия, дюймы, бедра, дюймы, бюст, сантиметры, Натуральная талия, сантиметры, Заниженная талия, сантиметры, Бедра, сантиметры, Длина стопы, мм, Торс, дюймы, США, Канада, Великобритания, Европа, континентальный, Россия, Япония, Франция, Австралия, Мексика, Китай, Корея ..

Размеры мужской одежды и обуви : мужские рубашки, мужские брюки / брюки, размер мужской обуви, размер букв, шея, дюймы, грудь, дюймы, рукав, дюймы, талия, дюймы, шея, сантиметры, грудь, сантиметры, Рукав, сантиметры, Талия, сантиметры, Длина стопы, мм, Длина стопы, дюймы, США, Канада, Великобритания, Австралия, Европа, континентальный, Япония, Россия, Франция, Италия, Испания, Китай, Корея, Мексика…

Механика

Преобразователь угловой скорости и частоты вращения : радиан / секунда, радиан / день, радиан / час, радиан / минута, градус / день, градус / час, градус / минута, градус / секунда, оборот / день, оборот / час, оборот / минута, оборот / секунда, оборот / год, оборот / месяц, оборот / неделя, градус / год, градус / месяц, градус / неделя, радиан / год, радиан / месяц, радиан / неделя.

Преобразователь ускорения : дециметр / секунда², метр / секунда², километр / секунда², гектометр / секунда², декаметр / секунда², сантиметр / секунда², миллиметр / секунда², микрометр / секунда², нанометр / секунда², пикометр / секунда², фемтометр / секунда² , аттометр / секунда², галлон, галилей, миля / секунда², ярд / секунда², фут / секунда², дюйм / секунда², ускорение свободного падения, ускорение свободного падения на Солнце, ускорение свободного падения на Меркурии, ускорение свободного падения на Венере , ускорение свободного падения на Луне, ускорение свободного падения на Марсе, ускорение свободного падения на Юпитере, ускорение свободного падения на Сатурне…

Конвертер плотности : килограмм / метр³, килограмм / сантиметр³, грамм / метр³, грамм / сантиметр³, грамм / миллиметр³, миллиграмм / метр³, миллиграмм / сантиметр³, миллиграмм / миллиметр³, экзаграмма / литр, петаграмм / литр, тераграмма / литр, гигаграмм / литр, мегаграмм / литр, килограмм / литр, гектограмм / литр, декаграмм / литр, грамм / литр, дециграмм / литр, сантиграмм / литр, миллиграмм / литр, микрограмм / литр, нанограмм / литр, пикограмм / литр , фемтограмм / литр, аттограмм / литр, фунт / дюйм³ …

Конвертер удельного объема : метр³ / килограмм, сантиметр³ / грамм, литр / килограмм, литр / грамм, фут³ / килограмм, фут³ / фунт, галлон (США ) / фунт, галлон (Великобритания) / фунт.

Преобразователь момента инерции : килограмм-метр², килограмм-сантиметр², килограмм-миллиметр², грамм-сантиметр², грамм-миллиметр², килограмм-сила-метр-секунда², унция-дюйм², унция-сила-дюйм-секунда², фунт-фут², фунт-сила-фут-секунда², фунт-дюйм². , фунт-сила-дюйм-секунда², ударный фут².

Конвертер момента силы : метр ньютон, метр килоньютон, метр миллиньютон, метр микроньютон, метр тонна-сила (короткий), метр тонна-сила (длинный), метр тонна-сила (метрический), метр килограмм-сила, грамм-сила-сантиметр, фунт-сила-фут, фунт-фут, фунт-дюйм.

Гидротрансформатор : ньютон-метр, ньютон-сантиметр, ньютон-миллиметр, килоньютон-метр, дин-метр, дин-сантиметр, дин-миллиметр, килограмм-сила-метр, килограмм-сила-сантиметр, килограмм-сила-миллиметр, грамм-сила-метр, грамм- сила-сантиметр, грамм-сила-миллиметр, унция-сила-фут, унция-сила-дюйм, фунт-сила-фут, фунт-сила-дюйм.

Термодинамика — тепло

Конвертер удельной энергии, теплоты сгорания (на массу) : джоуль / килограмм, килоджоуль / килограмм, калория (IT) / грамм, калория (th) / грамм, британские тепловые единицы (IT) / фунт, BTU (th) / фунт, килограмм / джоуль, килограмм / килоджоуль, грамм / калория (IT), грамм / калория (th), фунт / BTU (IT), фунт / Btu (th), фунт / лошадиная сила-час, грамм / лошадиная сила (метрическая) -час, грамм / киловатт-час.

Конвертер удельной энергии, теплоты сгорания (на объем) : джоуль / метр³, джоуль / литр, мегаджоуль / метр³, килоджоуль / метр³, килокалория (IT) / метр³, калория (IT) / сантиметр³, терм / фут³, терм / галлон (Великобритания), британские тепловые единицы (IT) / фут³, британские тепловые единицы на фут³, CHU / фут³, метр³ / джоуль, литр / джоуль, галлон (США) / лошадиная сила-час, галлон (США) / лошадиная сила (метрическая система) )-час.

Конвертер теплопроводности : ватт / метр / K, ватт / сантиметр / ° C, киловатт / метр / K, калория (IT) / секунда / сантиметр / ° C, калория (th) / секунда / сантиметр / ° C , килокалория (IT) / час / метр / ° C, килокалория (th) / час / метр / ° C, BTU (IT) дюйм / секунда / фут² / ° F, BTU (th) дюйм / секунда / фут² / ° F , BTU (IT) фут / час / фут² / ° F, Btu (th) фут / час / фут² / ° F, BTU (IT) дюйм / час / фут² / ° F, BTU (th) дюйм / час / фут² / ° F.

Конвертер удельной теплоемкости : джоуль / килограмм / K, джоуль / килограмм / ° C, джоуль / грамм / ° C, килоджоуль / килограмм / K, килоджоуль / килограмм / ° C, калория (IT) / грамм / ° C, калория (IT) / грамм / ° F, калория (th) / грамм / ° C, килокалория (IT) / килограмм / ° C, килокалория (th) / килограмм / ° C, килокалория (IT) / килограмм / K , килокалория (th) / килограмм / K, килограмм-сила-метр / килограмм / K, фунт-сила-фут / фунт / ° R, Btu (IT) / фунт / ° F, Btu (th) / фунт / ° F, Btu (IT) / фунт / ° R, Btu (th) / фунт / ° R, Btu (IT) / фунт / ° C, CHU / фунт / ° C.

Конвертер плотности теплового потока : ватт / метр², киловатт / метр², ватт / сантиметр², ватт / дюйм², джоуль / секунда / метр², килокалория (IT) / час / метр², килокалория (IT) / час / фут², калория (IT) / минута / сантиметр², калория (IT) / час / сантиметр², калория (th) / минута / сантиметр², калория (th) / час / сантиметр², дина / час / сантиметр, эрг / час / миллиметр², фут-фунт / минута на фут², лошадиные силы на фут², лошадиные силы (метрические единицы) на фут², британские тепловые единицы (IT) / секунда на фут², британские тепловые единицы (IT) в минуту на фут², британские тепловые единицы (ИТ) на час / фут², британские тепловые единицы (единицы) / секунда на дюйм² , Btu (th) / секунда / фут², Btu (th) / минута / фут², Btu (th) / час / фут², CHU / час / фут².

Преобразователь коэффициента теплопередачи : ватт / метр² / K, ватт / метр² / ° C, джоуль / секунда / метр² / K, килокалория (IT) / час / метр² / ° C, килокалория (IT) / час / фут² / ° C, BTU (IT) / секунда / фут² / ° F, Btu (th) / секунда / фут² / ° F, BTU (IT) / час / фут² / ° F, BTU (th) / час / фут² / ° F, CHU / час / фут² / ° C.

Гидравлика — жидкости

Конвертер объемного расхода : метр³ / секунда, метр³ / день, метр³ / час, метр³ / минута, сантиметр³ / день, сантиметр³ / час, сантиметр³ / минуту, сантиметр³ / секунда, литр / день, литр в час, литр в минуту, литр в секунду, миллилитр в день, миллилитр в час, миллилитр в минуту, миллилитр в секунду, галлон (США) в день, галлон (США) в час, галлон (США) в минуту, галлон (США) в секунду, галлон (Великобритания) в день, галлон (Великобритания) в час, галлон (Великобритания) в минуту, галлон (Великобритания) в секунду, килобаррель (США) в день, баррель (США) в день…

Конвертер массового расхода : килограмм / секунда, грамм / секунда, грамм / минута, грамм / час, грамм / день, миллиграмм / минута, миллиграмм / час, миллиграмм / день, килограмм / минута, килограмм / час , килограмм / день, экзаграмм / секунда, петаграмма / секунда, тераграмма / секунда, гигаграмма / секунда, мегаграмм / секунда, гектограмм / секунда, декаграмма / секунда, дециграмма / секунда, сантиграмма / секунда, миллиграмм / секунда, микрограмм / секунда, тонна (метрическая) / секунда, тонна (метрическая) / минута, тонна (метрическая) / час, тонна (метрическая) / день …

Конвертер молярной скорости потока : моль / секунда, экзамен / секунда, петамоль / секунда, терамоль / секунда, гигамоль / секунда, мегамоль / секунда, киломоль / секунда, гектомоль / секунда, декамоль / секунда, децимоль / секунда, сантимоль / секунда, миллимоль / секунда, микромоль / секунда, наномоль / секунда, пикомоль / секунда, фемтомоль / секунда, аттомоль в секунду, моль в минуту, моль в час, моль в день, миллимоль в минуту, миллимоль в час, миллимоль в день, километр в минуту, километр в час, километр в день.

Mass Flux Converter : грамм / секунда / метр², килограмм / час / метр², килограмм / час / фут², килограмм / секунда / метр², грамм / секунда / сантиметр², фунт / час / фут², фунт / секунда / фут².

Конвертер молярной концентрации : моль / метр³, моль / литр, моль / сантиметр³, моль / миллиметр³, километр / метр³, километр / литр, километр / сантиметр³, километр / миллиметр³, миллимоль / метр³, миллимоль / литр, миллимоль / сантиметр³, миллимоль / миллиметр³, моль / дециметр³, молярный, миллимолярный, микромолярный, наномолярный, пикомолярный, фемтомолярный, аттомолярный, зептомолярный, йоктомолярный.

Конвертер массовой концентрации в растворе : килограмм / литр, грамм / литр, миллиграмм / литр, часть / миллион, гран / галлон (США), гран / галлон (Великобритания), фунт / галлон (США), фунт / галлон (Великобритания), фунт / миллион галлон (США), фунт / миллион галлон (Великобритания), фунт / фут³, килограмм / метр³, грамм / 100 мл.

Конвертер динамической (абсолютной) вязкости : паскаль-секунда, килограмм-сила-секунда на метр², ньютон-секунда на метр², миллиньютон-секунда на квадратный метр, дин-секунда на сантиметр², равновесие, эксапуаз, петапуаз, терапуаз, гигапуаз, мегапуаз, килопуаз, гектопуаз, декапуаз, деципуаз, сантипуаз, миллипуаз, микропуаз, наноуаз, пикопуаз, фемтопуаз, аттопуаз, фунт-сила-секунда / дюйм², фунт-сила-секунда / фут², фунт-секунда / фут², грамм / сантиметр / секунда…

Конвертер кинематической вязкости : метр² / секунда, метр² / час, сантиметр² / секунда, миллиметр² / секунда, фут² / секунда, фут² / час, дюйм² / секунда, стоксы, экзастоки, петастоксы, терастоки, гигастоксы, мегастоксы, килостоки, гектостоки, декастоки, децистоки, сантистоки, миллистоки, микростоки, наностоки, пикостоки, фемтостоки, аттостоки.

Преобразователь поверхностного натяжения : ньютон на метр, миллиньютон на метр, грамм-сила на сантиметр, дина на сантиметр, эрг / сантиметр², эрг / миллиметр², фунт на дюйм, фунт-сила / дюйм.

Акустика — Звук

Преобразователь чувствительности микрофона : децибел относительно 1 вольт на 1 паскаль, децибел относительно 1 вольта на 1 микропаскаль, децибел относительно 1 вольта на 1 дин на квадратный сантиметр, децибел относительно 1 вольта на 1 микробар, вольт на паскаль, милливольт на паскаль, микровольт на паскаль.

Преобразователь уровня звукового давления (SPL) : ньютон на квадратный метр, паскаль, миллипаскаль, микропаскаль, дин / квадратный сантиметр, бар, миллибар, микробар, уровень звукового давления в децибелах.

Фотометрия — свет

Конвертер яркости : кандела на метр², кандела на сантиметр², кандела на фут², кандела на дюйм², килокандела на метр², стильб, люмен на метр² на стерадиан, люмен на сантиметр² на стерадиан, люмен на фут² стерадиан, нит, миллинит, ламберт, миллиламберт, фут-ламберт, апостиль, блондель, брил, скот.

Конвертер силы света : кандела, свеча (немецкий язык), свеча (Великобритания), десятичная свеча, свеча (пентан), пентановая свеча (мощность 10 свечей), свеча Хефнера, единица измерения яркости, десятичный буж, люмен / стерадиан, свеча (Международный).

Конвертер освещенности : люкс, метр-свеча, сантиметр-свеча, фут-свеча, фот, nox, кандела стерадиан на метр², люмен на метр², люмен на сантиметр², люмен на фут², ватт на сантиметр² (при 555 нм) .

Преобразователь частоты и длины волны : герцы, экзагерцы, петагерцы, терагерцы, гигагерцы, мегагерцы, килогерцы, гектогерцы, декагерцы, децигерцы, сантигерцы, миллигерцы, микрогерцы, единицы, микрогерцы / наногерцы, микрогерцы / секунды , длина волны в петаметрах, длина волны в тераметрах, длина волны в гигаметрах, длина волны в мегаметрах, длина волны в километрах, длина волны в гектометрах, длина волны в декаметрах…

Конвертер оптической силы (диоптрии) в фокусное расстояние : Оптическая сила (диоптрическая сила или преломляющая сила) линзы или другой оптической системы — это степень, в которой система сходится или рассеивает свет. Он рассчитывается как величина, обратная фокусному расстоянию оптической системы, и измеряется в обратных метрах в СИ или, чаще, в диоптриях (1 диоптрия = м⁻¹)

Электротехника

Конвертер электрического заряда : кулон, мегакулон , килокулон, милликулон, микрокулон, нанокулон, пикокулон, абкулон, EMU заряда, статкулон, ESU заряда, франклин, ампер-час, миллиампер-час, ампер-минута, ампер-секунда, фарадей (на основе углерода 12), элементарный плата.

Преобразователь электрического тока : ампер, килоампер, миллиампер, биот, абампер, ЭДС тока, статампер, ЭДС тока, СГС э.м. единица, CGS e.s. единица, микроампер, наноампер, ток Планка.

Линейный преобразователь плотности тока : ампер / метр, ампер / сантиметр, ампер / дюйм, абампер / метр, абампер / сантиметр, абампер / дюйм, эрстед, гильберт / сантиметр, ампер / миллиметр, миллиампер / метр, миллиампер / дециметр , миллиампер / сантиметр, миллиампер / миллиметр, микроампер / метр, микроампер / дециметр, микроампер / сантиметр, микроампер / миллиметр.

Преобразователь поверхностной плотности тока : ампер на метр², ампер на сантиметр², ампер на дюйм², ампер на мил², ампер на круговой мил, абампер на сантиметр², ампер на миллиметр², миллиампер на миллиметр², микроампер на миллиметр², миллиметр на милю миллиампер / сантиметр², микроампер / сантиметр², килоампер / сантиметр², ампер / дециметр², миллиампер / дециметр², микроампер / дециметр², килоампер / дециметр².

Преобразователь напряженности электрического поля : вольт на метр, киловольт на метр, киловольт на сантиметр, вольт на сантиметр, милливольт на метр, микровольт на метр, киловольт на дюйм, вольт на дюйм, вольт на мил, абвольт на сантиметр, статвольт / сантиметр, статвольт / дюйм, ньютон / кулон, вольт / микрон.

Преобразователь электрического потенциала и напряжения : вольт, милливольт, микровольт, нановольт, пиковольт, киловольт, мегавольт, гигавольт, теравольт, ватт / ампер, абвольт, EMU электрического потенциала, статвольт, ESU электрического потенциала, планковское напряжение.

Преобразователь электрического сопротивления : Ом, мегаом, микром, вольт / ампер, обратный сименс, абом, EMU сопротивления, статом, ESU сопротивления, квантованное сопротивление Холла, импеданс Планка, миллиом, кОм.

Преобразователь электрического сопротивления : омметр, ом-сантиметр, ом-дюйм, микром-сантиметр, микром-дюйм, ом-сантиметр, статом-сантиметр, круговой мил-ом / фут, ом-кв.миллиметр на метр.

Преобразователь электрической проводимости : сименс, мегасименс, килосименс, миллисименс, микросименс, ампер / вольт, mho, gemmho, micromho, abmho, statmho, квантованная проводимость Холла.

Конвертер электропроводности : сименс / метр, пикосименс / метр, mho / метр, mho / сантиметр, abmho / метр, abmho / сантиметр, статмо / метр, статмо / сантиметр, сименс / сантиметр, миллисименс / метр, миллисименс / сантиметр, микросименс / метр, микросименс / сантиметр, единица электропроводности, коэффициент проводимости, доли на миллион, шкала 700, шкала частей на миллион, шкала 500, частей на миллион, шкала 640, TDS, частей на миллион, шкала 640, TDS, части на миллион, шкала 550, TDS, частей на миллион, шкала 500, TDS, частей на миллион, шкала 700.

Преобразователь емкости : фарад, экзафарад, петафарад, терафарад, гигафарад, мегафарад, килофарад, гектофарад, декафарад, децифарад, сантифарад, миллифарад, микрофарад, емкость, нанофарад, аттофарад, фе , статфарад, ЭСУ емкости.

Преобразователь индуктивности : генри, экзагенри, петагенри, терагенри, гигагенри, мегагенри, килогенри, гектогенри, декагенри, децигенри, сантигенри, миллигенри, микрогенри, наногенри, пикогенри, индуктивность, фемогенри, атогенри , статенри, ЭСУ индуктивности.

Преобразователь реактивной мощности переменного тока : реактивный вольт-ампер, реактивный милливольт-ампер, реактивный киловольт-ампер, реактивный мегавольт-ампер, реактивный гигавольт-ампер.

American Wire Gauge Converter : Американский калибр проволоки (AWG) — это стандартизированная система калибра проводов, используемая в США и Канаде для измерения диаметров цветных электропроводящих проводов, включая медь и алюминий. Чем больше площадь поперечного сечения провода, тем выше его допустимая нагрузка по току.Чем больше номер AWG, также называемый калибром провода, тем меньше физический размер провода. Самый большой размер AWG — 0000 (4/0), а самый маленький — 40. В этой таблице перечислены размеры и сопротивление AWG для медных проводников. Используйте закон Ома для расчета падения напряжения на проводнике.

Магнитостатика, магнетизм и электромагнетизм

Преобразователь магнитного потока : Вебер, Милливебер, Микровебер, вольт-секунда, единичный полюс, мегалин, килолин, линия, максвелл, тесла-метр², тесла-сантиметр², гаусс-сантиметр², квант магнитного потока.

Конвертер плотности магнитного потока : тесла, Вебер / метр², Вебер / сантиметр², Вебер / дюйм², Максвелл / метр², Максвелл / сантиметр², Максвелл / дюйм², Гаусс, линия / сантиметр², линия / дюйм², гамма.

Радиация и радиология

Конвертер мощности поглощенной дозы излучения, общей мощности дозы ионизирующего излучения : серый / секунда, эксагрей / секунда, петагрей / секунда, терагрей / секунда, гигагрей / секунда, мегагрей / секунда, килограмм / секунда, гектоград / секунда, декаграй / секунда, дециграй / секунда, сантигрей / секунда, миллиграй / секунда, микрогрей / секунда, наногрей / секунда, пикграй / секунда, фемтогрей / секунда, аттогрей / секунда, рад / секунда, джоуль / килограмм / секунда, ватт на килограмм, зиверт в секунду, миллизиверт в год, миллизиверт в час, микрозиверт в час, бэр в секунду, рентген в час…

Радиоактивность. Конвертер радиоактивного распада : беккерель, петабеккерель, терабеккерель, гигабеккерель, мегабеккерель, килобеккерель, миллибеккерель, кюри, килокюри, милликюри, микрокюри, нанокюри, пикокюри, резерфорд, одна / секунда, дезинтеграции / секунда.

Конвертер облучения : кулон на килограмм, милликулон на килограмм, микрокулон на килограмм, рентген, миллирентген, микрорентген, тканевый рентген, Паркер, респ.

Радиация. Конвертер поглощенной дозы : рад, миллирад, джоуль / килограмм, джоуль / грамм, джоуль / сантиграм, джоуль / миллиграмм, серый, эксагрей, петагрей, терагрей, гигагрей, мегагрей, килограмм, гектагрей, декаграй, дециграй, сантигрей, микрогрей, миллиграй , наногрей, пикограй, фемтогрей, аттогрей, зиверт, миллизиверт, микрозиверт …

Прочие преобразователи

Конвертер метрических префиксов : нет, yotta, zetta, exa, peta, tera, giga, mega, kilo, hecto, deka , деци, санти, милли, микро, нано, пико, фемто, атто, зепто, йокто.

Преобразователь передачи данных : бит / секунда, байт / секунда, килобит / секунда (SI по умолчанию), килобайт / секунда (SI по умолчанию), кибибит / секунда, кибибайт / секунда, мегабит / секунда (SI по умолчанию) , мегабайт в секунду (SI по умолчанию), мебибит в секунду, мебибайт в секунду, гигабит в секунду (SI по умолчанию), гигабайт в секунду (SI по умолчанию), гибибит в секунду, гибибит в секунду, терабит в секунду (SI по умолчанию). .), терабайт в секунду (SI по умолчанию), тебибит в секунду, тебибайт в секунду, Ethernet, Ethernet (быстрый), Ethernet (гигабит), OC1, OC3, OC12, OC24, OC48 …

Типографика и цифровой Конвертер единиц изображения : твип, метр, сантиметр, миллиметр, символ (X), символ (Y), пиксель (X), пиксель (Y), дюйм, пика (компьютер), пика (принтер), точка (DTP / PostScript) ), point (компьютер), point (принтер), en, cicero, em, Didot point.

Конвертер единиц измерения объема пиломатериалов : кубический метр, кубический фут, кубический дюйм, футы для досок, тысяча футов для досок, шнур, шнур (80 фут3), футы для шнура, узел, поддон, поперечина, стяжка.

Калькулятор молярной массы : Молярная масса — это физическое свойство, которое определяется как масса вещества, деленная на его количество в молях. Другими словами, это масса одного моля определенного вещества.

Периодическая таблица : Периодическая таблица представляет собой список всех химических элементов, расположенных слева направо и сверху вниз по их атомным номерам, электронным конфигурациям и повторяющимся химическим свойствам, расположенным в форме таблицы таким образом, чтобы элементы с аналогичные химические свойства отображаются в вертикальных столбцах, называемых группами.У некоторых групп есть имена, а также номера. Например, все элементы группы 1, кроме водорода, являются щелочными металлами, а элементы группы 18 — благородными газами, которые ранее назывались инертными газами. Различные строки таблицы называются периодами, потому что это расположение отражает периодическое повторение сходных химических и физических свойств химических элементов по мере увеличения их атомного номера. Элементы одного периода имеют одинаковое количество электронных оболочек.

У вас есть трудности с переводом единицы измерения на другой язык? Помощь доступна! Задайте свой вопрос в TCTerms , и вы получите ответ от опытных технических переводчиков в считанные минуты.

Основы тормозных расстояний

Одна из самых важных частей автомобильного образования любого ученика водителя — это изучение тормозного пути транспортного средства. Если вы слишком внимательно следите за другими автомобилями, действие, также известное как прыгание сзади, является одной из основных причин автомобильных аварий в США, и выполнение этого действия во время вашего теста, безусловно, приведет к вашей неудаче.

Чтобы убедиться, что у вас есть четкое представление о тормозном пути и о том, почему они являются важной частью безопасности во время вождения, вот наше руководство по основам тормозного пути.

Что такое тормозной путь?

Тормозной путь транспортного средства — это комбинация «пути размышления» его водителя и фактического «тормозного пути».

На эти два расстояния могут влиять внешние факторы, такие как погода, дорожные условия, усталость водителя или состояние алкогольного опьянения.

Чем быстрее вы путешествуете, тем длиннее будет тормозной путь. Если вы едете со скоростью 40 миль в час, а не 30 миль в час, вам понадобится еще 13 метров, что больше трех средних значений длины автомобиля, чтобы остановиться.

Чтобы упростить задачу, ниже приведен список средних значений тормозного пути и тормозного пути для семейного автомобиля среднего размера в нормальных погодных условиях.

Среднее расстояние мышления

«Расстояние размышления» — это время, которое требуется водителю транспортного средства, чтобы заметить опасность и отреагировать на нее, нажав на тормоз.

Скорость Дистанция мышления (до реакции)
20 миль / ч 6 метров
30 миль / ч 9 метров
40 миль / ч 12 метров
50 миль / ч 15 метров
60 миль / ч 18 метров
70 миль / ч 21 метр

Средний тормозной путь

Тормозной путь — это расстояние, на которое ваша машина едет, когда вы нажимаете ногу на тормоз.Фактический тормозной путь вашего автомобиля зависит от его веса и дорожных условий. Например, небольшой городской автомобиль на сухой дороге остановится быстрее, чем пикап на мокром бетоне.

Скорость Тормозной путь
20 миль / ч 6 метров
30 миль / ч 14 метров
40 миль / ч 24 метра
50 миль / ч 38 метров
60 миль / ч 55 метров
70 миль / ч 75 метров

Как запоминать тормозной путь

Скорее всего, во время теста у вас не будет времени достать ноутбук и посмотреть эти таблицы, но не волнуйтесь, есть математическая формула, которую вы можете использовать для расчета тормозного пути.

Прежде чем застонать, успокойтесь, это на самом деле довольно просто. Все, что вам нужно сделать, это умножить скорость на 0,5, начиная с 2, чтобы получить тормозной путь в футах:

Если вам действительно нужно указать тормозной путь в метрах, просто разделите расстояние в футах на 3.3, чтобы получить тормозной путь в метрах. Это не так просто, поэтому вам может понадобиться калькулятор.

Обучение специалистов

Северо-западная автошкола и школа дорожного движения предоставляют жителям Лас-Вегаса уроки вождения и дорожного движения под руководством опытных инструкторов. Все наши инструкторы по вождению прошли проверку биографических данных, каждый автомобиль одобрен DMV по безопасности, и каждый член семьи Northwest стремится обеспечить отличное обучение водителей и обучение управлению рулем.

В Northwest вы можете рассчитывать найти выдающиеся классы, как в кампусе, так и за рулем, которые являются интересными, наполненными фактами, интересными и ориентированными на успех.

Добавить комментарий

Ваш адрес email не будет опубликован. Обязательные поля помечены *